cart-icon Товаров: 0 Сумма: 0 руб.
г. Нижний Тагил
ул. Карла Маркса, 44
8 (902) 500-55-04

Олимпиадные задания с ответами по литературе 6 класс: Олимпиада по литературе 6 класс, задания с ответами

Содержание

Олимпиада по литературе 6 класс, задания с ответами

На этой странице представлены примеры тестовых заданий и открытых вопросов олимпиады по литературе для 6 класса. Комплект заданий соответствует реальным олимпиадным заданиям и составлен в соответствии с действующей школьной программой.

Пользуясь заданиями, представленными на нашем сайте, учащиеся смогут подготовиться к участию в олимпиаде по литературе и сложат представление об уровне сложности заданий, с которыми им предстоит столкнуться. Для проверки внизу этой страницы составлены правильные ответы.

Скачайте задания, заполнив форму!

После того как укажете данные, кнопка скачивания станет активной

Тестовые задания

1. Найдите среди пословиц поговорку.
А) Сказывай тому, кто не знает Фому, а я родной брат ему.
Б) Худая та птица, которая гнездо своё марает.
В) Чужими руками жар загребать.
Г) Засыпь правду золотом, а она всплывёт

2. Не является произведением древнерусской литературы

А) «Сказание о белгородском киселе»
Б) «Песнь и вещем Олеге»
В) «Повесть о Петре и Февронии Муромских»
Г) «Повесть и разорении Рязани Батыем»

3. «Четвёртое лишнее»
А) «Свинья под дубом»
Б) «Листы и Корни»
В) «Муха»
Г) «Осёл и Соловей»

4. Жанр произведения А. С. Пушкина «Дубровский»
А) Роман
Б) Повесть
В) Притча
Г) Рассказ

5. Не является элементом композиции
А) Эпиграф
Б) Экспозиция
В) Развязка
Г) Кульминация

6. Этот поэт не принадлежит к числу поэтов XX века.
А) А. А. Ахматова
Б) Н. М. Рубцов
В) С. А. Есенин
Г) А. А. Фет

7. Автор повести «Уроки французского»
А) Ф. А. Искандер
Б) В. М. Шукшин
В) В. Г. Распутин
Г) В. П. Астафьев

8. Жанр произведения «Кладовая солнца»
А) Притча
Б) Повесть
В) Сказка-быль
Г) Рассказ

9. Найдите ошибку в определении понятия эпическое произведение.
А) Выражает душевное состояние, мысли автора о мире и человеке.

Б) Произведение художественной литературы
В) Рассказывает о людях, окружающем мире, событиях
Г) Жанры эпических произведений: роман, повесть, басня, сказка.

10. Жанр произведения А. С. Пушкина «Дубровский»
А) Роман
Б) Повесть
В) Притча
Г) Рассказ

Открытые вопросы

Вопрос 1
Назовите авторов рассказов.
«Галоша»
«Срезал»
«Тринадцатый подвиг Геракла»
«Конь с розовой гривой»

Вопрос 2
Кому из сказочных или литературных героев принадлежат перечисленные ниже предметы? Назовите героя, произведение и автора.
А) сабля и ранец
Б) волшебные волоски из бороды
В) ячменное семечко в цветочном горшке
Г) разбитое корыто
Д) говорящее чудесное зеркало

Вопрос 3
Укажите вид рифмовки.
И синий бесконечный лес
Скрывал ту сторону небес,
Куда, дневной окончив путь,
Уходит солнце отдохнуть.

Вопрос 4
Отгадайте загадки:
А)Шел долговяз, в сыру землю увяз
Б) Скатерть бела весь свет одела.
В)До неба достанет, а от земли не видать

Вопрос 5
Продолжите предложение: «Лирический герой – это…»

Ответы на тесты

Тестовое задание№ 1№ 2№ 3№ 4№ 5
ОтветВБВАА
Тестовое задание№ 6№ 7№ 8№ 9№ 10
ОтветГАВАА

Ответы на открытие вопросы

Ответ на вопрос 1: М. Зощенко, В. Шукшин, Ф. Искандер, В. Астафьев.
Ответ на вопрос 2:
А) сабля и ранец – солдат, сказка Г.-Х. Андерсена «Огниво»
Б) волшебные волоски из бороды — старик Хоттабыч, из одноимённой сказки Лагина
В) ячменное семечко в цветочном горшке – из него родилась Дюймовочка, сказка Г.-Х. Андерсена «Дюймовочка»

Г) разбитое корыто – старуха, А. С. Пушкин «Сказка о золотой рыбке»
Д) говорящее чудесное зеркало – мачеха, А. С. Пушкин «сказка о мёртвой царевне…»
Ответ на вопрос 3: Рифма – парная
Ответ на вопрос 4:
А) Дождь
Б) Снег
В) Дождь
Ответ на вопрос 5: Лирический герой — это образ того героя в лирическом произведении, переживания, мысли и чувства которого отражены в нём

Скачайте задания, заполнив форму!

После того как укажете данные, кнопка скачивания станет активной

Другие классы
Обновлено: , автор: Валерия Токарева

Олимпиада по литературе для 6 класса

Автор Оксана Анатольевна Лисовская,

учитель русского языка и литературы МБОУ СОШ № 82 города Ульяновска

Название учебно-методического комплекса: УМК по литературе Т. Ф. Курдюмовой для 6 класса

Цели и задачи: Проверить знания учащихся по пройденному материалу, воспитывать интерес к литературе, развивать память.

Оборудование и материалы: раздаточный материал

Вопросы:

  1. Какие средства художественной выразительности использованы в отрывке:
  2. С горы бежит поток проворный,
    В лесу не молкнет птичий гам,
    И гам лесной, и шум нагорный –
    Всё вторит весело громам.

  3. Определите жанр каждого из произведений, отрывки из которых приведены:
  4. а) Спустя некоторое время отошел от жизни святой митрополит Алексий, и снова блаженный Сергий был принуждаем с мольбою великодержавными князьями и всеми людьми воспринять престол метрополии Российской.
    б) Голодная лисица заметила свесившуюся с лозы гроздь винограда и хотела было достать ее, но не могла. Ушла она и говорит: «Он еще не дозрел». Иной не может сделать что-либо из-за недостатка сил, а винит в этом случай.
    в) Любишь кататься, люби и саночки возить.

    г) Спереди пятачок, сзади крючок, посередине спинка, на спинке щетинка.

  5. Какой русский поэт и переводчик сам себя шутя называл «дядькой всех ведьм и чертей на Руси»?
  6. Кто автор этих произведений?
  7. а) «Светлана», «Людмила», «Спящая царевна», «Лесной царь».
    б) «Бесы», «Гробовщик», «Метель», «Евгений Онегин».

  8. С именем какого русского писателя связаны эти места:
  9. а) Михайловское, Царский Лицей, Москва.
    б) Петербург, Пятигорск, Москва.

  10. Каким размером написано большинство басен И. А. Крылова. Например:
  11. Послушай, я не кинусь в слезы:
    Мне шутка все твои угрозы.
    Что я стараюсь приобресть,
    То не в твоих руках хранится;
    А чем не можешь поделиться,
    Того не можешь и унесть.

  12. О какой картине идет речь? Кто является ее автором? С каким стихотворением перекликается сюжет картин?
  13. Январский день. Комната, в которой находятся трое дорогих друг другу людей, кажется, залита теплом и светом. В центре – вдохновенная фигура поэта. На заднем плане изображена старенькая женщина, она замерла, покоренная музыкой слова своего воспитанника. Справа изображен друг поэта, он не просто слушает – внимает. В его взгляде – восхищение, глубокий интерес. Оказавшийся вдруг «в глуши, во мраке заточенья», друг дал почувствовать поэту спасительность «любви и дружества».

  14. О каких писателях идет речь?
  15. а) Он был не только прекрасным поэтом, переводчиком, но и мудрым педагогом, воспитателем наследника престола, будущего императора Александра II. А.С. Пушкин назвал себя его учеником; когда из печати вышла поэма А.С. Пушкина «Руслан и Людмила», этот поэт подарил Пушкину портрет с надписью «Победителю ученику от побежденного учителя».
    б) У себя дома, в имении Ясная Поляна под Тулой, он создает школу для крестьянских детей и сам учительствует в ней, пишет «Азбуку», создает четыре «Русские книги для чтения». К тому времени он уже вернулся с Кавказа и из Севастополя, где шла Крымская война. Он – один из самых знаменитых писателей России, в собрании его сочинений более девяноста томов.

    в) Студенту Петербургского университета всего 18 лет, он постеснялся отнести рукопись своей сказки в редакцию и отдал ее своему любимому профессору, а тот не только прочитал ее студентам, но и помог опубликовать в самом читаемом журнале «Библиотека для чтения». А.С. Пушкин после прочтения этой сказки сказал: «Теперь этот род сочинений можно мне и оставить».

  16. Придумайте монолог Ветра, Дождя, Старого Дома, Цветочной Вазы, Испорченного Телефона, Перегоревшей Лампочки, Домашнего Компьютера.

Ответы

  1. Олицетворение, эпитет, инверсия
  2. 1. Житие 2. Басня 3. Пословица 4. Загадка
  3. В.А.Жуковский
  4. 1. Жуковский 2. Пушкин
  5. 1. Пушкин 2. Лермонтов
  6. Четырехстопный ямб
  7. Н. Н. Ге «Пущин в гостях у Пушкина». Послание «И. И. Пущину»
  8. В.А. Жуковский, Л.Н. Толстой, П.П. Ершов

Олимпиадные задания по литературе 6 класс

      Олимпиадные задания по литературе для 6 класса

Задание 1.

Дайте определение понятия летописи как жанра литературы.


Задание 2.

Что такое «жанр».


Задание 3.

«Сказка-ложь, да в ней  намёк! Добрым молодцам урок!» — так закончил одну из своих сказок А.С.Пушкин. Приходилось ли вам извлекать «урок» из сказок. Если да, то какой?


Задание 4.

Продолжите предложение: «Лирический герой – это…»


Задание 5.

Назовите средства художественной выразительности, которые используются в данном отрывке

Вечор, ты помнишь, вьюга злилась,

а мутном небе мгла носилась;

Луна, как бледное пятно,

Сквозь тучи мрачные желтела,

И ты печальная сидела…


Задание 6.

Указать имя мифологического героя

             А) Иван-царевич

             Б) Геракл

             В) Артемида

             Г) Мельпомена


Задание 7.

Объясните устойчивые словосочетания из древнегреческой мифологии:

              А) Сизифов труд

              Б) Титанические усилия

              В) Олимпийское спокойствие


Задание 8.

Поставь ударения и определи размер стихотворения

Мороз и солнце; день чудесный.          

Еще ты дремлешь, друг прелестный, —

Пора, красавица, проснись:

Открой сомкнуты негой взоры…


Задание 9.

Прочитайте стихотворение А.Н. Плещеева «Весна». Ответьте на вопросы:

1. Какое изобразительно – выразительное средство лежит в основе стихотворения
( сравнение, эпитет, олицетворение).

2. Выпишите слова, которые подчёркивают эмоциональное состояние поэта, напиши какое.

Уж тает снег, бегут ручьи,

В окно повеяло весной…

Засвищут скоро соловьи,

И лес оденется листвою!

Чиста небесная лазурь,

Теплей и ярче солнце стало,

Пора метелей злых и бурь

Опять надолго миновала.

И сердце сильно таки в груди

Стучит, как будто ждёт чего-то,

Как будто счастье впереди

И унесла зима заботы!.


Задание 10.

Какие согласные звуки слышатся в следующей фразе:
Во втором часу ночи мы с братом были на вокзале.


Задание 11.

Расставьте ударения в следующих словах:
статуя, призывник, магазин, ненавидеть, средства, инструмент, километр, столяр, шофер, договор, квартал, звонишь.


Задание 12.

Выполните морфемный анализ следующих слов:
подземный, стройка, пришкольный, сумочка, выкупалась.


Задание 13.

Подберите 2 группы слов (в каждой по 2-3 слова) так, чтобы у тех и других был корень вод-, но с разным значением.


Задание 14.

Объясните смысловое различие между словами:
села – сёла, осел – осёл, поем – поём, смел – смёл, небо – нёбо.


Олимпиада по литературе 6 класс:              1 вариант    |       2 вариант    |       3 вариант

Олимпиада по литературе. 6 класс.

  1. 1. Соотнесите  жанровое определение с названиями произведений, приведенными  ниже

1

Летопись

 

А

«Лошадиная фамилия»

2

Басня

 

Б

«Кубок»

3

Баллада

 

В

«Повесть  временных  лет»

4

Сказка

 

Г

«Каша из  топора»

5

Рассказ

 

Д

«Мартышка и очки»

6

Стихотворение

 

Е

«Белая  береза»

Максимальное количество  баллов-6

2.Дайте определение загадки.

Отгадайте загадки:

А)Шел  долговяз, в сыру землю  увяз

Б) Скатерть  бела весь  свет одела.

В)До  неба достанет, а от  земли  не видать

Максимальное количество  баллов-4

3.Закончите пословицы:

А)Красна птица перьем..

Б)Где жить.…

В) Слово серебро…

Г) Сто  человек-

Максимальное количество  баллов-4

4.Русские художники написали  картины к русским  народным  сказкам: «Снегурочка», «Алёнушка», «Иван-царевич на сером волке», «Витязь на распутье», «Царевна», «Пёрышко Финиста – Ясного сокола», «Царевна-лягушка», «Сестрица Алёнушка», «Белая уточка», «Сказка об Иване-царевиче, Жар-птице и сером волке». Назовите фамилии этих художников.

Максимальное количество  баллов-2

5.Найдите ошибки  в определении литературоведческого термина басня.

Басня – жанр лирической прозы, в котором содержится открытое обличение действительности. Жанр возник в 19 веке  и нашел отражение в творчестве М.Ю. Лермонтова и Ф.И.Тютчева.

Максимальное количество  баллов- 4

7. Приведите из басен И.А.Крылова выражения, ставшие пословицами. Укажите, из каких они басен.

Максимальное количество  баллов – по  0.5 балла, если  указана пословица, по  одному  баллу, если  указано  название басни

 

8. Определите, каким стихотворным размером написано стихотворение А.С.Пушкина:

Буря мглою небо кроет,

Вихри снежные крутя;

То, как зверь, она завоет,

То заплачет, как дитя…

Максимальное количество  баллов — 1

 

9. Определите писателя по биографическим фактам.

 

А)

Родился в Москве, его  дядя Василий Львович был  известным  поэтом, на мальчика оказала большое влияние няня, учился в Царскосельском лицее, В.Жуковский подарил  ему свой портрет с надписью «Победителю  ученику от  побежденного  учителя»

Б)

Воспитывала его  бабушка, учился в Московском университете, поступил  в школу гвардейских  подпрапорщиков и кавалерийских  юнкеров, за  одно  свое произведение был  сослан  на Кавказ, погиб  на дуэли.

В)

Большую часть  своей жизни  провел  в Ясной Поляне, участвовал  в Крымской войне, открыл  школу для крестьянских  ребятишек, про  него  говорят, что он «гордость  русского  народа»

 

Максимальное количество баллов – 3

 

10. Ответьте на каждый из поставленных вопросов «да» (если утверждение верно) или «нет» (если утверждение неверно).

 

а)    В.Жуковского  называют  «солнцем русской поэзии»

б) Летописание на Руси началось  в XI веке.

В) Стихотворение, в основе которого  лежит  историческое событие, предание с острым  сюжетом называется балладой

Г) эпиграф – короткий текст, помещенный автором после произведения

Д) Повесть и слово — жанры древнерусской литературы.

 

Максимальное количество  баллов — 5

 

11. Придумайте монолог Ветра, Дождя, Старого Дома, Цветочной Вазы, Испорченного Телефона, Перегоревшей Лампочки, Незавёрнутого Крана, Домашнего Компьютера.

Максимальное количество  баллов-10

Всероссийская олимпиада школьников

Информационный раздел на сайте ГБУ ДО Центр «Интеллект», посвященный Всероссийской олимпиаде

Информационный портал Всероссийской олимпиады школьников

В 2018 году в заключительном этапе Всероссийской олимпиады школьников от Ленинградской области приняло участие 34 человека, из которых 4 приняли участие в нескольких олимпиадах. По результатам олимпиады 2 школьника Ленинградской области стали победителями и 9 — призерами (в 2017 году от Ленинградской области было 7 призеров).

Банк лучших олимпиадных заданий муниципального этапа Всероссийской олимпиады школьников


Физика

7 класс — задания

7 класс — ответы

8 класс — задания

8 класс — ответы

9 класс — задания

9 класс — ответы

10 класс — задания

10 класс — ответы

11 класс — задания

11 класс — ответы


Экология

Задания

Ответы


География

Задания

Ответы


Обществознание

7 класс — задания

7 класс — ответы

8 класс — задания

8 класс — ответы

9 класс — задания

9 класс — ответы

10 класс — задания

10 класс — ответы

11 класс — задания

11 класс — ответы


История

7 класс — задания

7 класс — ответы

8 класс — задания

8 класс — ответы

9 класс — задания

9 класс — ответы

10-11 классы — задания

10-11 классы — ответы


Мировая художественная культура

5-6 классы — задания

5-6 — ответы

7-8 классы — задания

7-8 классы — ответы

9 класс — задания

9 класс — ответы

10 класс — задания

10 класс — ответы

11 класс — задания

11 класс — ответы


Экономика

5-7 классы — задания

5-7 классы — ответы

8-9 классы — задания

8-9 классы — ответы

10-11 классы — задания

10-11 классы — ответы


ОБЖ

7-8 классы — задания

7-8 классы — ответы

9 класс — задания

9 класс — ответы

10-11 классы — задания

10-11 классы — ответы


Физическая культура

7-8 классы — задания

7-8 классы — ответы

9-11 классы — задания

9-11 классы — ответы


Английский язык

7-8 классы — задания

7-8 классы — ответы

9-11 классы — задания

9-11 классы — ответы


Испанский язык

Задания

Ответы


Немецкий язык

7-8 классы — задания

7-8 классы — бланк ответов

7-8 классы — ответы

9-11 классы — аудирование

9-11 классы — аудирование — бланк ответа

9-11 классы — чтение

9-11 классы — чтение — бланк ответа

9-11 классы — страноведение

9-11 классы — страноведение — бланк ответа

9-11 классы — письмо

9-11 классы — письмо — бланк ответа

9-11 классы — говорение

9-11 классы — лексико-грамматическое задание

9-11 классы — лексико-грамматическое задание — бланк ответа

9-11 классы — ответы


Литература

7 класс — задания

8 класс — задания

9 класс — задания

10 класс — задания

11 класс — задания

Ответы к заданиям


Русский язык

7-8 классы — задания

7-8 классы — ответы

9 класс — задания

9 класс — ответы

10-11 классы — задания

10-11 классы — ответы


Олимпиады и конкурсы

 

2020-2021 учебный год

Приказы

О проведении регионального этапа всероссийской и заключительного этапа республиканской олимпиад школьников в Республике Татарстан в 2020-2021 учебном году

Приказ № под-954/20 от 10.09.20 МО и Н РТ о проведении школьного этапа олимпиад. 

Приказ № 503 от 16 сентября 2020г. в Заинском муниципальном районео проведении школьного этапа олимпиад.

График проведения школьного этапа 

Согласие на использование и обработку персональных данных.

Олимпиадные задания:

Астрономия, экономика, право, математика 4-7, математика 8-11, экология, русский язык, МХК, история, обществознание, литература, география, ОБЖ (теория), ОБЖ (практика), биология, физика, физическая культура (теогрия), физическая культура (практика), английский язык (аудирование), химия, геология, технология (теория),  История Татарстана, русский язык для учащихся школ с родным языком обучения, русская литература для учащихся школ с родным языком обучения, родной (татарский) язык для учащихся-татар школ с русским языком обучения, татарский язык для учащихся русскоязычных групп школ с русским языком обучения, родная (татарская) литература для учащихся школ с родным языком обучения, родная (татарская) литература для учащихся школ с русским языком обучения, родной (татарский) язык для учащихся школ с родным языком обучения.

Ключи к олимпиадным заданиям: 

Астрономия, экономика, право, математика, экология, русский язык, МХК, история, обществознание, литература, география, ОБЖ (теория), биология, физика, физическая культура (теория), английский язык, химия, геология, технология, История Татарстана, русский язык для учащихся школ с родным языком обучения, русская литература для учащихся школ с родным языком обучения, родной (татарский) язык для учащихся-татар школ с русским языком обучения, татарский язык для учащихся русскоязычных групп школ с русским языком обучения, родная (татарская) литература для учащихся школ с родным языком обучения, родная (татарская) литература для учащихся школ с русским языком обучения, родной (татарский) язык для учащихся школ с родным языком обучения.

Первичный протокол: астрономия, экономика, право, математика, экология, русский язык, МХК, история, обществознание, литература, география, ОБЖ, биология, физика, физическая культура (Д), физическая культура (Ю),  английский язык, химия, геология, технология (юноши), технология (девушки), История Татарстана, русский язык для учащихся школ с родным языком обучения, русская литература для учащихся школ с родным языком обучения, родной (татарский) язык для учащихся-татар школ с русским языком обучения, татарский язык для учащихся русскоязычных групп школ с русским языком обучения, родная (татарская) литература для учащихся школ с родным языком обучения, родная (татарская) литература для учащихся школ с русским языком обучения, родной (татарский) язык для учащихся школ с родным языком обучения.

Итоговый протокол: астрономия, экономика, право, математика, экология, русский язык, МХК, история, обществознание, литература, география, ОБЖ, биология, физика, физическая культура (Д), физическая культура  (Ю), английский язык, химия, геология, технология (юноши), технология (девушки), История Татарстана, русский язык для учащихся школ с родным языком обучения, русская литература для учащихся школ с родным языком обучения, родной (татарский) языкдля учащихся-татар школ с русским языком обучения, татарский язык для учащихся русскоязычных групп школ с русским языком обучения, родная (татарская) литература для учащихся школ с родным языком обучения, родная (татарская) литература для учащихся школ с русским языком обучения, родной (татарский) язык для учащихся школ с родным языком обучения.

 2019-2020 учебный год

Приказы

 

 

 

Приказ №1957/19 о проведении заключительного этапа республиканских олимпиад школьников «Путь к Олимпу» среди школьников 6-11кл. 2019-2010 у.г.

Сроки публикации первичного и итогового протоколов 

Олимпиадные задания:

Астрономия,  экология, английский язык (письменный тур, устный тур), география, искусство МХК,  русский язык, математика 4-7, 8-11, история, биология, ОБЖ (теория, практика), литература, физическая культура (теория, практика), обществознание, физика, химия, право (https://cloud.mail.ru/stock/nMzgdd3atY4xogKYMK77JxDE), экономика, русский язык для учащихся школ с родным языком обучения, родной (татарский) язык для учащихся школ с родным языком обучения, история Татарстана и татарского народа, родная (татарская) литература для учащихся школ с родным языком обучения, русская литература для учащихся школ с родным языком обучения, родной  (татарский) язык для учащихся школ с русским языком обучения, геология, родная (татарская) литература для учащихся-татар школ с русским языком обучения, татарский язык для учащихся русскоязычных групп школ с русским языком обучения.

Ключи к олимпиадным заданиям:

Астрономия,  экология, английский язык (письменный тур), география, искусство МХК, русский язык  МР, математика, история, биология, ОБЖ (теория, практика), литература, физическая культура, обществознание, физика, химия, право (https://cloud.mail.ru/stock/iZzwDADia98gdMVX4y5TWdjY), экономика, русский язык для учащихся школ с родным языком обучения, родной (татарский) язык для учащихся школ с родным языком обучения, история Татарстана и татарского народа, родная (татарская) литература для учащихся школ с родным языком обучения, русская литература для учащихся школ с родным языком обучения, родной  (татарский) язык для учащихся школ с русским языком обучения, геология, родная (татарская) литература для учащихся-татар школ с русским языком обучения, татарский язык для учащихся русскоязычных групп школ с русским языком обучения.

 

Первичный протокол:

Астрономия, информатика, английский язык, география, искусство (МХК), русский язык, математика, экология, история, биология, ОБЖ, литература, физическая культура (девушки, юноши), обществознание, физика, химия, право, технология (девушки, юноши), экономика, русский язык для учащихся школ с родным языком обучения, родной (татарский) язык для учащихся школ с родным языком обучения, история Татарстана, родная (татарская) литература для учащихся школ с родным языком обучения, русская литература для учащихся школ с родным языком обучения, родной (татарский) язык для учащихся-татар школ с русским языком обучения, геология, родная (татарская) литература для учащихся-татар школ с русским языком обучения, татарский язык для учащихся русскоязычных групп школ с русским языком обучения.

Итоговый протокол:

Астрономия, информатика, английский язык, география, искусство МХК, русский язык, математика, экология, история, биология, ОБЖ, литература, физическая культура (юноши, девушки), обществознание, физика, химия, право, технология (юноши, девушки), экономика, русский язык для учащихся школ с родным языком обучения, родной (татарский) язык для учащихся школ с родным языком обучения, история Татарстана, родная (татарская) литература для учащихся школ с родным языком обучения, русская литература для учащихся школ с родным языком обучения, родной (татарский) язык для учащихся-татар школ с русским языком обучения, геология, родная (татарская) литература для учащихся-татар школ с руссуким языком обучения, татарский язык для учащихся русскоязычных групп школ с русским языком обучения.

Конкурсы 2018-2019 учебный год

13.12.2018_41337_Udachina G.L._Makarov V.V       О проведении XIX Открытого республиканского телевизионного молодежного фестиваля эстрадного искусства «Созвездие – Йолдызлык»

 

Олимпиады 2018-2019 учебный год

Приказы

Приказ МО и Н РТ от 06.09.2018г. №под-1314/18  «О проведении школьного этапа всероссийской и республиканской олимпиад школьников в Республике Татарстан в 2018-2019 учебном году»

О проведении школьного этапа всероссийской и республиканской олимпиад школьников в 2018-2019 учебном году

Приказ МО и Н РТ  от 26.10.2018г. №под-1580/18 О проведении муниципального этпа всероссийской и республиканской олимпиад школьников в 2018/2019 учебном году

Задания

русский язык,   экология,   история,  английский язык (письменный тур) , английский язык (устный тур) , география , искусство (МХК) , астрономия,

математика (4-7),    математика (8-11), биология,  ОБЖ (теория),  ОБЖ (практика),  литература,  технология (теория, практика), технология (защита проекта), обществознание, физика,  химия,  право,  физическая культура (теория,  практика),   русский язык для учащихся школ с родным (нерусским) языком обучения,  татарский язык для школ с татарским языком обучения, История Татарстана и татарского народа,   татарская литература для учащихся школ с татарским языком обучения, русская литература для учащихся школ с родным (нерусским) языком обучения,  татарский язык для учащихся-татар школ с русским языком обучения,   геология,   татарская литература для учащихся-татар школ с русским языком обучения,   татарский язык для учащихся русскоязычных групп школ с русским языком обучения,   экономика.

Ключи к олимпиадным заданиям

русский язык,  экология,  история,  английский язык ,география, астрономия,

математика, МХК, биология,  ОБЖ,  литература,  обществознание, физика,  химия,   право,   физическая культура,  русский язык для учащихся школ с родным (нерусским) языком обучения,  татарский язык для школ с татарским языком обучения,   История Татарстана и татарского народа,  татарская литература для учащихся школ с татарским языком обучения,  русская литература для учащихся школ с родным (нерусским) языком обучения,  татарский язык для учащихся-татар школ с русским языком обучения,  геология,   татарская литература для учащихся-татар школ с русским языком обучения,  татарский язык для учащихся русскоязычных групп школ с русским языком обучения, экономика.

Протоколы

Итоговый протокол (русский язык) ,   экология,   история, информатика,  английский язык,   географмя,  МХК,  астрономия,   математика,   биология, ОБЖ, литература, технология (мальчики, девочки),  обществознание,   физика, химия,  право,  физическая культура (девушки),  физическая культура (юноши),  русский язык для учащихся школ с родным (нерусским языком обучения),  татарский язык для учащихся школ с татарским языком обучения,   история Татарстана и татарского народа, татарская литература для учащихся школ с татарским языком обучения,  русская литература для учащихся школ с родным (нерусским) языом обучения,  ИП татарский язык для учащихся татар школ с русским языком обучения ,  геология, татарская литература для учащихся — татар школ с русским языком обучения,  татарский язык для учащихся русскоязычных групп школ с русским языком обучения, экономика. 

 

 

Олимпиады 2017-2018 учебный год

Приказы

Письмо МОиН РТ ГАОУ «Республиканский олимпиадный центр» от 10.01.2018г. №206 «О проведении регионального этапа всероссийской и заключительного этапа республиканской олимпиад школьников в Республике Татарстан в 2017-2018 учебном году»

Приказ МОиН РТ от 15.12.2017г. №под-2046/17 «О проведении регионального этапа всероссийской и заключительного этапа республиканской олимпиад школьников в Республике Татарстан в 2017/2018 учебном году»

О проведении муниципального этапа всероссийской и республиканской олимпиад школьников в Республике Татарстан в 2017/2018 учебном году

О проведении школьного этапа всероссийской и республиканской олимпиад школьников в Республике Татарстан в 2017/2018 учебном году

Задания олимпиадные

русский язык, экология, история, физика, англ.язык (письм.тур),  англ.язык (устн.тур),  астрономия, МХК, география, математика, биология, ОБЖ (теорет.тур), ОБЖ (практ.тур), литература, технология (теория), технология (практика), обществознание, химия, право, физическая культура, русский язык для учащихся школ с родным (нерусским) языком обучения, татарский язык для учащихся школ с татарским языком обучения, история Татарстана и татарского народа, татарская литература для учащихся школ с татарским языком обучения, русская литература для учащихся школ с родным (нерусским) языком обучения, экономика, татарский язык для учащихся-татар школ с русским языком обучения, геология, татарская литература для учащихся-татар школ с русским языком обучения, татарский язык для учащихся русскоязычных групп школ с русским языком обучения

Ключи к олимпиадным заданиям

русский язык, экология, история, физика, англ.язык,  астрономия, МХК, география, математика, биология, ОБЖ, литература, технология, обществознание, химия, право, физическая культура, русский язык для учащихся школ с родным (нерусским) языком обучения, татарский язык для учащихся школ с татарским языком обучения, история Татарстана и татарского народа, татарская литература для учащихся школ с татарским языком обучения, русская литература для учащихся школ с родным (нерусским) языком обучения, экономика, татарский язык для учащихся-татар школ с русским языком обучения, геология, татарская литература для учащихся-татар школ с русским языком обучения, татарский язык для учащихся русскоязычных групп школ с русским языком обучения    

Протоколы

русский язык, экология, история, физика, англ.язык, астрономия, искусство (МХК), география, математика, биология, ОБЖ, литература, технология, обществознание,  информатика и ИКТ, химия, физическая культура, право, русский язык для учащихся школ с родным (нерусским) языком обучения, татарский язык для учащихся школ с татарским языком обучения, история Татарстана и татарского народа, татарская литература для учащихся школ с татарским языком обучения, русская литература для учащихся школ с родным (нерусским) языком обучения, экономика, татарский язык для учащихся-татар школ с русским языком обучения, геология, татарская литература для учащихся-татар школ с русским языком обучения, татарский язык для учащихся русскоязычных групп школ с русским языком обучения

Конкурсы 2017-2018 учебный год

Конкурсы для школьников на получение премий, стипендий, грантов в 2018 году (скачать)

Конкурсы для педагогов на получение премий, стипендий, грантов в 2018 году (скачать)

Методический комплекс «Моя родословная» (рабочая тетрадь для внеклассной работы 5-6 классов) (скачать)

Конкурс презентаций «Конституция Республики Татарстан глазами детей», «Мои конституционные права и обязанности», «Я — гражданин многонациональной республики» среди учащихся 1-4 классов общеобразовательных организаций Республики Татарстан; конкурс интервью по темам: «Конституция Республики Татарстан в жизни гражданина» и «Роль Конституции Республики Татарстан в жизни государства» среди учащихся 5-9 классов общеобразовательных учреждений Республики Татарстан (скачать)

Олимпиада 2016-2017 учебный год

 

Олимпиадные задания:

экология, математика, история, география, английский (письменный тур), (устный тур), биология, физика, русский язык, экономика, МХК, ОБЖ (теория, практика), химия, обществознание, технология, литература, право, астрономия, физическая культура (теория, практика), русский язык для учащихся школ с родным (нерусским) языком обучения, татарский язык для учащихся школ с татарским языком обучения, История Татарстана и татарского норода, татарская литература для учащихся школ с татарским языком обучения, русская литература для учащихся школ с родным (нерусским) языком обучения, татарский язык для учащихся-татар школ с русским языком обучения, геология, татарская литература для учащихся-татар школ с русским языком обучения, татарский язык для учащихся русскоязычных групп школ с русским языком обучения.

 

Ключи к олимпиадным заданиям:

экология, математика, история, география, английский язык, биология, физика, русский язык, экономика, МХК, ОБЖ (теория, практика), химия, обществознание, технология, литература, право, астрономия, физическая культура, русский язык для учащихся школ с родным (нерусским) языком обучения, татарский язык для учащихся школ с татарским языком обучения, История Татарстана и татарского народа, татарская литература для учащихся школ с татарским языком обучения, русская литература для учащихся школ с родным (нерусским) языком обучения, татарский язык для учащихся-татар школ с русским языком обучения, геология, татарская литература для учащихся-татар школ с русским языком обучения, татарский язык для учащихся русскоязычных групп школ с русским языком обучения.

 

Протоколы:

Протоколы по экологии, математике, истории, географии, английскому языку, биологии, физике, русскому язку, экономике, МХК, ОБЖ, химии, обществознанию, ИКТ, технологии (мальчики, девочки), литературе, праву, астрономии, физической культуре (девушки, юноши), русский язык для учащихся школ с родным (нерусским) языком обучения, татарский язык для учащихся школ с татарским языком обучения, история Татарстана и татарского народа, татарская литература для учащихся школ с татарским языком обучения, русская литература для учащихся школ с родным (нерусским) языком обучения, татарский язык для учащихся-татар школ с русским языком обучения, геология, татарская литература для учащихся-татар школ с русским языком обучения, татарский язык для учащихся русскоязычных групп школ с русским языком обучения.

 

Приказы:

ВОШ, РОШ

 

Олимпиада 2015-2016 учебный год

Приказ «О проведении школьного этапа всероссийской и республиканской олимпиады школьников в 2015/2016 уч.г.» (скачать)

Приказ «О проведении муниципального этапа всероссийской и республиканской олимпиады школьников в 2015-2016 учебном году» (скачать)

Методические рекомендации по проведению школьного и муниципального этапов всероссийской олимпиады

 

Олимпиадные задания

Олимпиадные задания: по русскому языку для учащихся школ с родным (нерусским) языком обучения, русской литературе для учащихся школ с родным (нерусским) языком обучения, геологии, истории Татарстана, экологии,  английскому языку (письменный тур), английскому языку (устный тур),   астрономии, русскому языку,    физике, немецкому языку (письменный тур), немецкому языку (устный тур), математике, биологии, истории, ОБЖ (теория), ОБЖ (практика), обществознанию, химии, географии, физической культуре (практика), физическая культура (теория), литература, технология (теория), технология (практика), экономика, право, тат. яз. НШ, тат. лит НШ, тат. яз. РШ, тат. лит. РШ, тат. яз. (русскоязычные группы).

 

Ключи к олимпиадным заданиям

Ключи по русскому языку для учащихся школ с родным (нерусским) языком обучения, русской литературе для учащихся школ с родным (нерусским) языком обучения, геологии, истории Татарстана, экологии, английскому языку (письменный тур), английскому языку (устный тур),   астрономии,  русскому языку, физике, немецкому языку (письменный тур), математике, биологии, истории, ОБЖ, обществознанию, химии, географии, физической культуре, литература, технологии, экономике, праву, тат. яз. НШ, тат. лит. НШ, тат. яз. РШ, тат. лит. РШ, тат. яз. (русскоязычные группы).

 

Протоколы

Протокол по русскому языку для учащихся школ с родным (нерусским) языком обучения, русской литературе для учащихся школ с родным (нерусским) языком обучения,  геологии, истории Татарстана, экологии, английскому языку, астрономии, русскому языку, физике, немецкому языку, математике, биологии,  истории, МХК, ОБЖ, обществознанию, химии, географии, физической культуре, литературе РШ, технологии, экономике, праву, тат. яз. НШ, тат лит. НШ, тат. яз. РШ, тат. лит. РШ, тат. яз. (русскоязычные группы).

 

Приказы по итогам МЭ олимпиад

ВОШ, РОШ

 

Конкурсы
Молодежные образовательные проекты для учащейся молодежи «Молодежные интеллектуалы России»

IX Всероссийский конкурс научно-инновационных проектов для старшеклассников (скачать)

Ежегодная Межрегиональная олимпиада школьников (скачать)

Олимпиада 2014-2015 учебный год

Приказ  «Об установлении сроков проведения регионального этапа всероссийской олимпиады школьников по общеобразовательным предметам в 2014/15 уч.г.» (скачать)

Приказ «О проведении заключительного этапа всероссийской олимпиады школьников в 2014/15 уч.г.» (скачать)

График проведения регионального этапа всероссийской олимпиады школьников и республиканской олимпиады для 8-х классов по предметам, включённым в состав всероссийской олимпиады школьников в 2014/2015 учебном году (скачать)

Олимпиадные задания

Олимпиадные задания:  по геологии,    экологии,   английскому языку (письменный тур),  английскому языку (устный тур),   астрономии,   биологии,   экономике,  немецкому языку,  математике 4-6 кл.,  математике 7-11 кл.,   русскому языку,   истории,  ОБЖ (практика), ОБЖ (теория),   обществознанию,  МХК, инструкция для Участника муниципального тура олимпиады по информатике,  физической культуре (практика),   физической культуре (теория),    химии,  географии,  праву,  технологии, физике, литературе, тат.языку для тат.школ,   тат. лит. для школ с тат. яз. обуч.,  тат языку для учащихся школ с русским языком обучения, татарской литературе для учащихся школ с русским языком обучения, татарский язык для учащихся русскоязычных групп школ с русским языком обучения, русскому языку для учащихся школ с родным (нерусским) языком обучения, русской литературе для учащихся школ с родным (нерусским) обучения

 

Ключи к олимпиадным заданиям

Ключи по геологии,   экологии,   английскому языку (письменный тур),  астрономии,  биологии,  экономике,  немецкому языку,  математике,  русскому языку,  истории,  ОБЖ (теория), обществознанию,   химии,   МХК,   географии,   праву,   физической культуре,  технологии,  физике,  литературе,  тат.языку для тат.школ,     тат. лит. для учащихся школ с татарским языком обучения,  тат языку для учащихся школ с русским языком обучения,  татарской литературе для учащихся школ с русским языком обучения,  татарский язык для учащихся русскоязычных групп школ с русским языком обучения,  русскому языку для учащихся школ с родным (нерусским) языком обучения,  русской литературе для учащихся школ с родным (нерусским) обучения, 

 

Протоколы

Протоколы по геологии,    экологии,   английскому языку,   астрономии,    биологии,  экономике, немецкому языку,   математике,  русскому языку,  истории, ОБЖ, обществознанию, МХК,   химии,   географии,   праву,  физической культуре, технологии,   физике,    литературе (с рус яз обуч),  татарский язык для учащихся школ с татарским языком обучения,   татарской литературе для учащихся школ с татарским языком обучения, татарскому языку для учащихся-татар школ с русским языком обучения, татарской литературе для учащихся-татар школ с русским языком обучения, татарский язык для учащихся русскоязычных групп школ с русским языком обучения, русскому языку для учащихся школ с родным (нерусским) языком обучения, русской литературе для учащихся школ с родным (нерусским) обучения

 

Положение об организации и проведении олимпиады по теме «Защита прав потребителей» (скачать)

Олимпиада 2013 — 2014 учебный год

Методические рекомендации по организации и проведению школьного и муниципального туров олимпиады (скачать)

Протоколы муниципального тура Всероссийской олимпиады и республиканской олимпиады школьников

по экологии

по биологии

по химии

по английскому языку

по истории

по физической культуре

по геграфии

по обществознанию

по физике

по русской литературе

по экономике

по астрономии

по геологии

по математике

по праву

по русскому языку для русских школ

по МХК

по немецкому языку

по ОБЖ

по технологии

татарская литература для учащихся школ с татарским языком обучения

татарский язык для учащихся школ с татарским языком обучения

татарская литература среди учащихся татарских групп школ с обучением на русском языке

татарский язык среди учащихся татарских групп школ с обучением на русском языке

татарский язык среди учащихся русских групп школ с обучением на русском языке

русская литература для учащихся школ с татарским языком обучения

русский язык для учащихся школ с татарским языком обучения

Олимпиадные задания по предметам: экология, биология, химия, английский язык, история, физическая культура, география, обществознание, физика, русская литература, экономика, астрономия, геология, математика, право, русский язык для русских школ, МХК 8 класс, МХК 9 класс, МХК 10 класс (часть 1), МХК 10 класс (часть 2), МХК 11 класс, немецкий язык, ОБЖ, технология, татарская литература для татарских школ, татарский язык для татарских школ, татарская литература для русских школ(татарская группа), татарский язык для русских школ(татарская группа), татарский язык для русских школ(русская группа), русская литература для татарских школ, русский язык для татарских школ,

Решения  олимпиадных заданий по предметам: экология, биология, химия, английский язык, история, физическая культура, география, обществознание, физика, экономика, астрономия, геология, математика, право, русский язык для русских школ, МХК, немецкий язык, ОБЖ, технология, татарская литература для татарских школ, татарский язык для татарских школ, татарская литература для русских школ(татарская группа), татарский язык для русских школ(татарская группа), татарский язык для русских школ(русская группа), русская литература для татарских школ, русский язык для татарских школ,

 

Олимпиада 2012 — 2013 учебный год

О проведении школьного и муниципального этапа Всероссийской  олимпиады школьников  2012/2013 уч.г.

Протоколы муниципального тура Всероссийской олимпиады и республиканской олимпиады школьников

по русской литературе

по русскому языку

по астрономии

по математике

по информатике

по технологии

по биологии

по искусству

по ОБЖ

по физике

по английскому языку

по экологии

по географии

по немецкому языку

по экономике

история

право

физическая культура

общество

русский язык нац.

химия

русская литература нац.

татарский язык для учащихся школ с татарским языком обучения

татарская литература для учащихся школ с татарским языком обучения

татарский язык среди учащихся татарских групп школ с обучением на русском языке

татарская литература среди учащихся татарских групп с обучением на русском языке

татарский язык для учащихся русскоязычных групп школ с обучением на русском языке

 Олимпиадные задания по предметам:  математика,  русский язык,  литеретура,  технология,  ОБЖ,  искусство,  физика, биология,  информатика, физическая культура., английский язык, экология, география,экономик, астрономия, немецкий язык, история, общество, право, физическая культура, русский язык для татарских школ, русская литература для татарских школ, татарский язык для учащихся школ с татарским языком обучения, татарская литература для учащихся школ с татарским языком обучения, татарский язык среди учащихся татарских групп школ с обучением на русском языке, татарская литература среди учащихся татарских групп с обучением на русском языке, татарский язык и литература для учащихся русскоязычных групп школ с обучением на русском языке

Решения  олимпиадных заданий по предметам: математика, русский язык, литература, технология, ОБЖ, исскусство,  физика, английский язык, экология, география, экономика, астрономия, немецкий язык, биология, история, право, физическая культура,обществознание,химия, русский язык нац.,русская литература нац.

Разбор олимпиадных заданий


 

РУССКИЙ ЯЗЫК (19.09.2016г.)

 

4 класс                              5 класс                                   6 класс                             7 класс                              

 

8 класс                              9 класс                                   10 класс                           11 класс

 


 

МАТЕМАТИКА (20.09.2016г.)

 

4 – 11 класс (задания)                              4-11 класс (ответы)

 


 

БИОЛОГИЯ (21.09.2016г.)

 

6 класс                              7 класс                                   8 класс                         9 класс                        

 

10-11 классы          

         


 

ИСТОРИЯ (22.09.2016г.)

 

ЗАДАНИЯ                           5 класс                                   6 класс                             7 класс

 

8 класс                              9 класс                                   10 класс                           11 класс

 

 

ОТВЕТЫ                             5 класс                                   6 класс                             7 класс 

 

8 класс                              9 класс                                   10 класс                           11 класс

 


 

ЛИТЕРАТУРА (23.09.2016г.)

 

                      5 класс                                   6 класс                             7 класс

 

8 класс                              9 класс                                   10 класс                           11 класс

 


 

ОБЩЕСТВОЗНАНИЕ (26.09.2016г.)

 

ЗАДАНИЯ                           5 класс                                   6 класс                             7 класс

 

8 класс                              9 класс                                   10 класс                           11 класс

 

 

ОТВЕТЫ                             5 класс                                   6 класс                             7 класс 

 

8 класс                              9 класс                                   10 класс                           11 класс

 


 

ГЕОГРАФИЯ (27.09.2016г.)

 

                     6 класс                                     7 класс                                  8 класс

                  

9 класс                                 10 класс                                  11 класс

 


 

ИНФОРМАТИКА (28.09.2016г.)

 

9 – 11 класс (задания)                              9-11 класс (ответы)

 


 

МИРОВАЯ ХУДОЖЕСТВЕННАЯ КУЛЬТУРА (29.09.2016г.)

 

ЗАДАНИЯ                          

 

7-8 классы                              9 класс                                   10 класс                           11 класс

 

 

ОТВЕТЫ                             

 

7-8 классы                              9 класс                                   10 класс                           11 класс

 


 

ПРАВО (03.10.2016г.)

 

10 – 11 класс

 


 

Art of Problem Solving

The Art of Problem Solving размещает этот AoPSWiki, а также многие другие онлайн-ресурсы для студентов, интересующихся математическими олимпиадами. Посмотрите AoPSWiki. В отдельных статьях часто есть примеры проблем и решений для многих уровней решателей проблем. Многие также имеют ссылки на книги, веб-сайты и другие ресурсы, относящиеся к теме.

  • Учебники по математике
  • Математические форумы
  • Сайты по математике

Содержание

  • 1 Учебные сайты
  • 2 Бесплатная электронная книга математических формул и стратегий
  • 3 курса математики
  • 4 Список ресурсов
  • 5 классов для соревнований по математике
  • 6 задач для соревнований по математике
    • 6.1 Проблемные книги
    • 6.2 Проблемы в сети
      • 6.2.1 Вводные средства решения проблем
      • 6.2.2 Средство решения проблем
      • 6.2.3 Олимпиада по решению задач
  • 7 статей
  • 8 Огромный список ссылок
    • 8.1 Рекомендации курса AoPS
    • 8.2 AMC 8 Подготовка
      • 8.2.1 Проблемы
    • 8.3 AMC 10/12 Подготовка
      • 8.3.1 Проблемы
    • 8.4 Подготовка к AIME
      • 8.4.1 Проблемы
    • 8.5 Подготовка к началу олимпиады
      • 8.5.1 Связка общих ссылок
      • 8.5.2 Проблемы
    • 8.6 Подготовка к олимпиаде среднего / продвинутого уровня
      • 8.6.1 Проблемы
    • 8.7 Ссылки на книги:
      • 8.7.1 Уровень олимпиады
        • 8.7.1.1 Бесплатно
        • 8.7.1.2 Не бесплатно
    • 8.8 Наборы задач
  • 9 См. Также

Учебные сайты

  • AMC Problem Trainer: https: // amctrainer.com / play
  • Total Problem Trainer: https://vqbc.github.io/trivial/

Бесплатная электронная книга математических формул и стратегий

Более 130 страниц бесплатной электронной книги по математическим формулам и стратегиям: https://www.omegalearn.org/thebookofformulas

Курсы математики

Введение в теорию чисел: https://thepuzzlr.com/math-courses

Бесплатный учебный курс AMC 8, охватывающий все основные концепции: https://thepuzzlr.com/courses/amc-8-bootcamp/

Список ресурсов

Элементарно: https: // www.omegalearn.org/elementary-competition-math

Средний: https://www.omegalearn.org/middle-competition-math

Высокий

: https://www.omegalearn.org/high-competition-math

Все, что вам нужно знать об AMC 8: https://thepuzzlr.com/courses/amc-8-bootcamp/


Класс основ AMC 8: https://www.omegalearn.org/amc8-fundamentals

Плейлист AMC 8 Video Solutions: https://www.youtube.com/watch?v=TRGPF3BxujE&list=PLbhMrFqoXXwmwbk2CWeYOYPRbGtmdPUhL

Класс AMC / MATHCOUNTS: https: // www.omegalearn.org/amc8-advanced

Соревновательные классы по математике

  • «Искусство решения проблем» проводит уроки, пользующиеся популярностью среди многих наиболее успешных студентов в Соединенных Штатах. Серия проблем AoPS.

Задачи олимпиады по математике

Проблемные книги

На многих олимпиадах по математике продаются книги о прошлых олимпиадах и решениях. Эти книги могут быть отличным дополнительным материалом для заядлых студентов-математиков.

  • ARML содержит четыре сборника задач, охватывающих большую часть ARML, а также некоторые соревнования NYSML.Однако их, как правило, сложно найти. Некоторые можно заказать здесь.
  • книг MOEMS доступны здесь, на AoPS.
  • книг MATHCOUNTS доступны здесь, на AoPS.
  • книг AMC доступны здесь, на AoPS.
  • Книги
  • конкурса Мандельброта доступны здесь, на сайте AoPS.

Проблемы в сети

Art of Problem Solving поддерживает очень большую базу данных задач математических конкурсов. Многие веб-сайты математических конкурсов включают архивы прошлых задач.Список олимпиад по математике ведет к ссылкам на многие из домашних страниц этих соревнований. Вот несколько примеров:

Вводные решатели проблем
  • Mu Alpha Theta.org содержит прошлые задачи конкурса.
  • Noetic Learning Challenge Math — Решение задач для одаренных учеников начальной школы.
  • Страница упражнений Elias Saab по MathCounts.
  • Домашняя страница олимпиады школьников штата Алабама по математике.
  • Южноафриканская олимпиада по математике включает в себя многолетние прошлые проблемы с решениями.
  • Beestar.org — Еженедельные задания по решению проблем и рейтинг почета, 1–8 классы
Решение проблем среднего уровня
  • Задачи и решения конкурса по математике AoPS
  • Прошлые проблемы USAMTS можно найти на домашней странице USAMTS.
  • Еженедельные задачи Иваны Александровой по математике для старшеклассников содержат хорошие задачи, которые заставят вас задуматься и научат вас новым навыкам и материалам
  • Сайт Kalva — один из лучших ресурсов по математическим задачам на планете.(В настоящее время оффлайн. Зеркало находится на этой странице)
  • Прошлые задачи Колорадской математической олимпиады (CMO) можно найти на домашней странице CMO.
  • Прошлые задачи по международному поиску талантов (IMTS) можно найти здесь
  • Brilliant — это веб-сайт, на котором можно решать задачи, чтобы набирать очки и переходить на более высокий уровень.
  • Clevermath Аналогично предыдущему
Olympiad Problem Solvers
  • Задачи и решения конкурса по математике AoPS
  • Math and CS Research — это издание по математике и информатике со статьями и наборами задач по широкому кругу тем.
  • Прошлые проблемы USAMTS можно найти на домашней странице USAMTS.
  • Сайт Kalva — один из лучших ресурсов по математическим задачам на планете. (В настоящее время не в сети, но доступно несколько зеркал, например здесь.)
  • Математические головоломки Ника — сложные задачи с подсказками и решениями.
  • Канадская математическая олимпиада проводится здесь Канадским математическим обществом.
  • Задачи Всесоюзных олимпиад по математике 1961-1986 гг. — Проблем много, решений нет.[Сайт больше не существует. Сайт заменен веб-снимком]
  • Прошлые задачи по международному поиску талантов (IMTS) можно найти здесь
  • Olympiad Math Madness — Стеки сложных задач, без решения. [Сайт больше не существует. Сайт заменен веб-захватом]

Статьи

  • Управление временем
  • Плюсы и минусы математических соревнований Ричарда Руска.
  • «Создание позитивной культуры ожидания в математическом образовании», автор — Дэррил Хилл, лауреат премии «Сестра Схоластика».
  • «Хватит делать глупых ошибок», Ричард Рушик.
  • Какие вопросы на самом деле являются глупыми? Ричард Рушик.
  • Обучение через обучение
  • Ричард Рушик и Мэтью Кроуфорд «Как написать математическое решение».
  • Неравенства доктора Киран Кедлая
  • Неравенство на олимпиаде Томаса Дж. Милдорфа
  • Теория олимпиадных чисел: абстрактная перспектива Томаса Дж. Милдорфа
  • Теория чисел Наоки Сато
  • Теория олимпиадных чисел через сложные задачи, Джастин Стивенс
  • Барицентрические координаты в геометрии олимпиады Макс Шиндлер и Эван Чен
  • Повышение экспоненты (LTE), Амир Хоссейн Парварди
  • Метод uvw Матиаса Бока Тейса Кнудсена
  • Китайская теорема об остатках, Эван Чен
  • Contest Reflections by Wanlin Li

Огромный список ссылок

Рекомендации курса AoPS

  • Искусство решения проблем Рекомендации курса
  • Вы все еще не можете решить, какой курс? Перейдите по указанной выше ссылке и нажмите свяжитесь с нами в нижней части раздела «Карта курса», чтобы запросить личные рекомендации!

AMC 8 Подготовка

Бесплатные классы AMC 8: https: // thepuzzlr.ru / курсы / amc-8-bootcamp /

Проблемы

Бесплатные занятия AMC 8: omegalearn.org/amc8-fundamentals omegalearn.org/amc8-advanced

Эти классы охватывают все важные концепции, необходимые для успешной работы с AMC 8.

Решения AMC 8 для видео: https://www.youtube.com/watch?v=TRGPF3BxujE&list=PLbhMrFqoXXwmwbk2CWeYOYPRbGtmdPUhL

AMC 8 Проблемы в разделе ресурсов

Проблема и решения: Проблемы AMC 8 в вики AoPS

AMC 10/12 Подготовка

AMC 10/12 130+ страниц Книга математических формул и стратегий: https: // www.omegalearn.org/thebookofformulas

Бесплатные занятия AMC 10/12: omegalearn.org/amc10-12

Как подготовка к AIME поможет AMC 10/12 Score

Какой класс выбрать?

AMC 10 для практики AMC 12

AMC Prep

AMC 10/12 Подготовка

Перекрытие и подготовка AIME / AMC 10

Как подготовиться к amc10 и aime?

Подготовка к AMC 10?

Проблемы

AMC 10 Проблемы в разделе ресурсов

AMC 10 Проблемы в AoPS Wiki

AMC 12 Проблемы в разделе ресурсов

Проблемы с AHSME (старый AMC 12) в AoPS Wiki

AMC 12 Проблемы в AoPS Wiki

Препарат AIME

Учеба для получения права на USAMO

Как подготовиться к AIME

Подготовка к AIME

Использование вопросов, не относящихся к AIME, для подготовки к AIME

Лучшие книги для подготовки к AIME?

Как улучшить рейтинг AIME для создания JMO?

Подготовка к AIME и USAMO

Проблемы

Проблемы с AIME в разделе ресурсов

Проблемы с AIME в AoPS Wiki

Проблемы AIME, отсортированные по сложности

Подготовка к началу олимпиады

  • Общие
  • Общий
  • Как подготовиться к USAJMO?
  • Подготовка / решение USAMO
  • Более легкие олимпиады для практики USAJMO?
  • Для USAMO: ACoPS или Engel?
  • Олимпиадные задачи — как подготовиться
  • USAMO / Подготовка к олимпиадам: с чего начать?
  • USAJMO prepare
Связка общих ссылок
  • Подготовка к USAMO
  • Подготовка к USAMO
  • USAMO
  • Усамо приготовление
  • Подготовка к USAMO
  • Обратный отсчет до USAMO
  • Подготовка USAMO
  • USAJMO Prep
  • Подготовка USAMO
  • USAJMO Prep
  • Как подготовиться к USAMO / Making Red MOP
  • Жесткая подготовка
  • Подготовка к USAMO и JMO
  • USAMO PREP
  • Новичок в USAMO
  • Что мне делать?
  • Повышение до уровня USAMO и IMO
  • Подготовка для США (J) MO
  • соревнования по математике / несколько советов, как добиться хороших результатов
  • Подготовка к олимпиаде
  • Подготовка к олимпиаде
  • USAJMO Prep
  • Правильное обучение
  • Что ведет к успеху
Проблемы
  • Проблемы USAJMO в разделе ресурсов
  • Проблемы USAJMO в AoPS Wiki
  • Проблемы USAMO в разделе ресурсов
  • Задачи USAMO в AoPS Wiki

Подготовка к олимпиаде среднего и продвинутого уровней

Проблемы
  • Практическая олимпиада 1
  • Практическая олимпиада 2
  • Практическая олимпиада 3
  • Решения для практических олимпиад
  • Проблемы USAMO в разделе ресурсов
  • Проблемы USAMO в AoPS Wiki
  • Проблемы IMO в разделе ресурсов
  • Проблемы IMO в AoPS Wiki

Ссылки на книги:

Уровень олимпиады
Бесплатно
  • Леммы в олимпиаде по геометрии статья
  • Плоская геометрия
  • Эван Чен ОТИС-Выдержки
  • Основы олимпиадной теории чисел
  • Теория олимпиадных чисел через сложные задачи
  • Современная олимпиадная теория чисел
Несвободно
  • Плоская евклидова геометрия: теория и проблемы
  • Евклидова геометрия в математических олимпиадах
  • Комплексные числа и геометрия
  • Геометрия комплексных чисел
  • Комплексные числа от A до… Z
  • 103 Задачи тригонометрии: Из тренировок группы ИМО США
  • Введение в диофантовы уравнения: проблемно-ориентированный подход
  • Введение в теорию чисел и неравенства
  • 104 Задачи теории чисел: Из обучения группы ИМО США
  • 102 Комбинаторные задачи
  • Путь к комбинаторике для студентов: стратегии подсчета
  • -fkmr1 Математические олимпиады США 1972-1986 Проблемы и решения
  • Искусство и ремесло решения проблем
  • Стратегии решения проблем

Наборы задач

  • Сборник прошлых статей в формате PDF
  • Практические задачи со всего мира
  • Общие задачи олимпиадной математики
  • Набор задач условной вероятности
  • 31 олимпиадная задача по вероятностному методу
  • 567 Хорошее и жесткое неравенство
  • Неравенства
  • 100 полиномиальных задач
  • Задачи тригонометрии
  • Общие все уровни
  • Теория чисел
  • Задачи олимпиады
  • 33 Функциональные уравнения
  • Проблемы индукции
  • Индукционные решения
  • 260 Проблемы геометрии
  • 150 задач геометрии
  • 50 задач диофантовых уравнений
  • 60 задач геометрии
  • 116 Проблем
  • Алгебраические неравенства
  • 100 задач комбинаторики
  • 100 проблем
  • Теория чисел
  • Геометрия
  • Общий
  • 100 задач теории чисел
  • 100 задач функциональных уравнений
  • Начало / промежуточный счет и вероятность
  • 40 Функциональные уравнения
  • 100 Геометрических неравенств
  • 10 забавных нестандартных задач 🙂
  • 169 Функциональные уравнения
  • Геометрия треугольника
  • Вероятность
  • Алгебра
  • Теория чисел
  • Геометрия круга
  • Другая геометрия

Рейтинг всех олимпиад (уровень сложности)

См. Также

  • Список олимпиад по математике
  • Стипендии по математике
  • Научные соревнования
  • Соревнования по информатике
  • Как мне подготовиться

Международная экономическая олимпиада

4.1. Соревнование в бизнесе является командным и включает в себя устное представление
результатов. Презентации должны сопровождаться слайдами. Презентации должны быть на английском языке.

4.2. Примерная часть представляет собой командное соревнование и длится два дня — 1) день подготовки; 2) день презентации. В течение дня подготовки участники могут использовать любые онлайн и офлайн материалы, но запрещено обращаться за помощью к другим людям. Все команды отправляют свои слайды
до 23:59 по местному времени подготовительного дня.После этого времени вносить изменения в слайды нельзя.

4.3. Бизнес-кейс состоит из нескольких частей. Вот компетенции, которые должен приобрести конкурсант, которые будут оцениваться жюри по этим частям:

Аналитическое мышление: Способность структурно подойти к решению сложной бизнес-задачи, правильно разделив ее на потоки (на направления внутри в котором может лежать решение
проблемы).

Как правило, команда должна разбить дело на несколько больших блоков, которые, в свою очередь, далее разбиваются и так далее до уровня конкретных задач.Хорошая структура соответствует принципу MECE (взаимоисключающий, коллективно исчерпывающий), то есть она охватывает все возможные решения, но каждый отдельный поток не пересекается с другими.

Концептуальное мышление: способность строить правильные гипотезы на основе полученной структуры, созданной путем анализа. Здесь команда проверяет, как идеи реагируют на необходимый запрос и правильно решают те или иные проблемы предприятия, команда также следит за тем, чтобы эти решения были осуществимы и имеют здравый и деловой смысл.

Количественное мышление: Ни один случай не может быть решен без простых, но быстрых расчетов и более сложных моделей, которые иллюстрируют определенный анализ.

Коммуникативные навыки: умение правильно строить презентацию и отвечать на вопросы.

4.4. ПК рекомендует следующую литературу. Полезная литература для решения кейсов:

  • Принцип пирамиды, Барбара Минто;
  • Взломайте дело, Дэвид Орвалл;
  • Доверенный советник Д.Майстер, К. Грин, Р. Гэлфорд.

Полезная литература для анализа данных:
  • Путь Маккинси, Итан Расиэль;
  • Стратегический менеджмент, Томпсон Стрикленд;
  • Пятая дисциплина, Питер Сенге;
  • Мыслить быстро и медленно, Даниэль Канеман;
  • Дело в Пойнт, Марк Косентино;
  • BCG по стратегии, К. Стерн, М. Деймлер.

Полезная литература для презентаций:
  • Say It With Charts, Gene Zelazny;
  • Раскладывая салфетку, Дэн Роам;
  • Визуализируйте это, Натан Яу;
  • Принцип пирамиды, Барбара Минто;
  • Слайд: ology, Нэнси Дуарте

4.5. Помимо вышеперечисленных книг, ПК рекомендует изучить примерный кейс, который использовался в предыдущие годы.

США побеждают на «самой сложной» олимпиаде по математике | Математика

Это был последний вопрос Международной математической олимпиады (IMO), ежегодного «Кубка мира по математике» для учащихся среднего школьного возраста, проводившегося в этом году в Таиланде и завершившегося в среду победителями из США:

Дон Не паникуйте, если это оставит вас в недоумении. У некоторых из лучших математических умов мира тоже были проблемы.Руководитель британской группы IMO, доктор Джефф Смит из Университета Бата, сказал, что это была самая сложная работа в истории IMO, которая впервые была проведена в 1959 году.

Пороговая оценка для золотых медалей — которая меняется ежегодно в зависимости от того, насколько хорошо участники выполнить — был установлен на 26 баллов из 42, что является самым низким показателем за всю историю. Вырвав пять золотых медалей, США обыграли обычных победителей Китай. Британская команда из шести студентов, включая 16-летнего «математика» Джо Бентона, заняла 22-е место из 104 стран-участниц.

Смит сказал, что его команда была «довольна как удар» после того, как выиграла четыре серебряные медали. Член команды Уоррен Ли был на одно очко меньше.

Смит отметил, что Франция заняла 14-е место. «Почти всегда Великобритания финиширует выше Франции. В этом году malheureusement , ситуация обратная. Я поздравил французского лидера, упомянув, что он новый Наполеон ».

Экзамен проводится в течение двух дней подряд, и у участников есть четыре с половиной часа на решение трех задач в день, которые могут включать геометрию, теорию чисел и алгебру.Вам не нужны знания в области высшей математики, такой как математический анализ, но вопросы разработаны так, чтобы быть чрезвычайно сложными. Калькуляторы не допускаются.

В среду более 570 подростков из Афганистана и Эквадора стояли группами, расслабляясь после объявления результатов. Во время поездки некоторые катались на слонах или совершали походы в горы вокруг Чиангмая.

Майкл Курал, 17 лет, из Коннектикута, США, сказал, что он провел июнь в лагере с пятью членами своей команды в возрасте от 16 до 18 лет.«Это было определенно намного труднее, чем мы привыкли», — сказал подросток о конкурсе, когда его товарищи по команде играли на своих телефонах и обсуждали мероприятие за приготовленным на пару рисом и острым карри из свинины. «Я думаю, что многие команды не привыкли к этому».

Он сказал, что у них есть преимущество, потому что их тренер тренировал их с особенно сложными контрольными работами. «В первый же день нам удалось вырваться далеко вперед».

Када Уильямс, 16 лет, из Венгрии, разочарован 20-м местом своей команды.«Мы ожидали, что добьемся большего», — сказал он. «Исходя из этих низких пороговых значений для золотых медалей, все были не слишком хороши, что заставило нас чувствовать себя немного лучше».

Если вышеупомянутый вопрос слишком технический, попробуйте его из IMO 2011 года, который легче понять, если не решить:

Предположим, что вы отмечаете конечный набор точек на бесконечной плоскости таким образом, что вы не можете провести прямую линию через любые три отмеченные точки.Мы определяем ветряную мельницу как следующий процесс: провести на плоскости бесконечную прямую линию ровно через одну из отмеченных точек. Затем поверните линию по часовой стрелке, используя выбранную отмеченную точку в качестве точки поворота, пока движущаяся линия не коснется другой отмеченной точки. В этот момент новая отмеченная точка становится точкой поворота, и линия продолжает вращаться по часовой стрелке. Этот процесс продолжается, и время от времени появляются новые повороты.

Покажите, что можно выбрать одну из отмеченных точек и выбрать начальную линию через эту точку под определенным углом, чтобы получившаяся ветряная мельница использовала каждую отмеченную точку в качестве точки поворота бесконечно много раз.

Молодые, одаренные и сбитые с толку

На Международной математической олимпиаде соревнуются не более шести участников от страны. Эти подростки — лучшие в мире студенты-математики доуниверситетского образования. Помимо необычайных способностей, многие из них будут обучены, часто в течение многих лет, решать задачи олимпиадного стиля.

Олимпиада состоит из двух тестов в последовательные дни, каждый по четыре с половиной часа и каждый из трех вопросов. Вопросы начинаются с самых простых и постепенно усложняются.

Вопросы составлены таким образом, чтобы их мог легко понять любой человек, имеющий базовые представления о математике, и вопрос, показанный выше, безусловно, так. В нем нет сложных слов — последовательность означает просто список чисел, а целое число — это положительное или отрицательное целое число. Точно так же нет странных символов. ≤ и ≥ означают, что меньше или равно и больше или равно. Большой сигма-символ — это то, что должно быть известно специалистам A-level, и обозначает объединенную сумму следующих за ним терминов.

Можно понять, о чем идет речь в вопросе, даже не зная, как его решить. На олимпиадные вопросы нет простых ответов, иначе лучшие молодые математические умы не потратили бы на решение каждого по 90 минут. Или, в данном случае, не решать. В этом году 74 из 104 команд набрали нулевые баллы по вышеуказанному вопросу. Алекс Беллос

The Ultimate STEM Guide for Kids: 239 Cool Sites

Никогда не рано побуждать детей и подростков к получению образования и занятиям в области естественных наук, технологий, инженерии и математики (STEM).Важно подготовить будущие поколения к успеху, поскольку карьера в этих областях продолжает расти.

В этом руководстве представлены летние лагеря, веб-сайты, конкурсы, приложения и ресурсы по карьере для учащихся от начальной до средней школы. Гендерный разрыв в этих областях постепенно сокращается по мере появления новых инициатив, в том числе таких организаций, как Girls Who Code, но, поскольку женщины по-прежнему крайне недопредставлены, мы включили мероприятия STEM специально для девочек и молодых женщин.

Мы надеемся, что это руководство вдохновит детей и подростков использовать свои навыки решения проблем, чтобы найти ответы на невозможные задачи и помочь сформировать лучшее будущее.

STEM Fun для детей классы K-12 ↑

Классные веб-сайты STEM

  • Ask Dr. Universe: программа Ask Dr. Universe Университета штата Вашингтон позволяет детям изучать различные темы STEM и получать ответы на общие вопросы. Есть вопрос, которого нет на сайте? Отправьте его на их странице «Спросите»!
  • Code.org: Никто не слишком молод (или я мог бы добавить), чтобы писать код. Узнайте, как создать игру для iPhone, написать свою первую компьютерную программу, нарисовать JavaScript и многое другое.
  • Инженерное дело, дерзайте! (eGFI): откройте для себя гайки и болты инженерии.Этот веб-сайт содержит советы о карьере, развлекательную информацию по всем направлениям и ссылки на журнал eGFI.
  • Учащиеся EPA: Ищете новости об окружающей среде, ресурсы для домашних заданий, информацию о конкурсах или идеи для школьного проекта, ориентированного на окружающую среду? Посетите этот веб-сайт Агентства по охране окружающей среды.
  • Exploratorium: Один из моих любимых. Веб-сайт Exploratorium из Сан-Франциско забит интерактивными занятиями, видео, приложениями, ссылками и многим другим.
  • Extreme Science: Чрезвычайно интересно. Здесь вы найдете дикие и странные факты о природе, ресурсы для научных проектов и информацию о самых разных мировых рекордах.
  • How Stuff Works: Я посещаю этот сайт каждый день. В нем сотни и тысячи статей, объясняющих чудеса науки (и почти все остальное на планете).
  • Музей науки и промышленности Чикаго Интернет-наука: приложения, занятия и видео, о боже! Играйте в игры, наблюдайте за вылуплением цыплят, создавайте виртуальные химические реакции или используйте криминалистику для анализа различных типов конфет.
  • НАСА Образование для студентов: информация о карьере, галереи изображений, телевидение НАСА, тематические статьи и статьи… все, что вы хотите знать об аэрокосмической отрасли, вы обязательно найдете здесь.
  • Академия наук, инженерии, математики и аэрокосмической науки НАСА (SEMAA): SEMAA была разработана для увеличения участия молодежи K-12, которая исторически недостаточно обслуживалась, в областях STEM. Школьные мероприятия и летние сессии проводятся по всей стране.
  • NOVA: Веб-сайт научно-популярного шоу PBS переполнен видео и статьями.Исследуйте чудеса эволюции, природы, физики, математики — практически любой предмет STEM, который вам интересен.
  • Друзья науки: увязните в науке. На этом веб-сайте есть более 1000 идей для проектов научной ярмарки, руководства по проектам, комплекты проектов и подробные профили карьеры в STEM.
  • Science Channel: задавайте вопросы обо всем. На этом веб-сайте есть не только краткое изложение телевизионных программ канала Science Channel, но и множество видеороликов, викторин, игр и последних новостей науки.
  • STEM-Works: В дополнение к статьям и информации о вакансиях, STEM-Works снабдил свой сайт интересными мероприятиями.Проверьте свои навыки в викторине про рептилий. Спасите спортсмена в Бионических играх. Или просто следуйте по пути великих белых с Global Shark Tracker.
  • Virtual Tech Camps: круглогодичное онлайн-обучение для детей и подростков с учебной программой на Python, Java, Minecraft, Roblox, AI, Unreal Engine, Adobe, 3D-моделировании и многом другом. Сэкономьте 100 долларов с кодом ВМЕСТЕ.
  • Tynker: вычислительная платформа, которая позволяет детям развивать навыки программирования с помощью веселых творческих курсов. Присоединяйтесь к миллионам детей со всей страны, которые учатся программировать с Tynker!
  • Академия Ккан: Академия Хана разбирает сложные темы по ряду предметов, включая программирование, математику и статистику! У него есть учебные ресурсы почти по всем предметам STEM K-12.
  • Flite Test STEM: Flite Test предоставляет учащимся до 12 классов множество легко загружаемых бесплатных уроков, которые знакомят учащихся с STEM посредством авиационной деятельности.
  • HOODA MATH: HOODA MATH — это веб-сайт, посвященный математическим играм, разделенным по предметам. Учащиеся K-12 и любители математики любого возраста могут узнать о числах, развлекаясь.

Вызовы и конкурсы STEM

  • Задача Симена «Мы можем изменить мир»: у вас есть сила, чтобы спасти планету. В конкурсе Siemen по экологической устойчивости K-12 команды со всей страны соревнуются, чтобы улучшить свои сообщества.Множество призов.
  • Toshiba / NSTA ExploraVision: ExploraVision — уникальное научное соревнование для школьников до 12 лет. Команды из двух-четырех учеников работают с учителем, моделируя задачи реальных исследований и разработок.

Награды STEM

Ресурсы для карьеры в STEM

  • Бюро статистики труда K-12: Министерство труда США занято. Здесь вы найдете диаграммы, карты и многие другие ресурсы о карьере и экономике США.
  • WeUseMath.org: Вы когда-нибудь задумывались (как я часто задавал), когда вы собираетесь использовать математику в реальной жизни? На этом веб-сайте, посвященном карьере в математике, есть более чем несколько ответов.

Государственные инициативы в области STEM

  • Образовать для инноваций: запущенная в 2009 году программа «Образовать для инноваций» направлена ​​на то, чтобы вывести американских студентов из среднего звена на вершину в области естественных наук и математики. Это породило ряд федеральных усилий и благотворительных инициатив (см. Ниже).
  • STEM AmeriCorps: эта многолетняя инициатива направлена ​​на размещение членов AmeriCorps в некоммерческих организациях STEM (таких как FIRST) для работы в общинах с недостаточным уровнем обеспеченности услугами.
  • Ярмарка науки в Белом доме: на этой научной ярмарке президент выступает в роли хозяина! Студентов чествуют за новаторские проекты, разработки и эксперименты, пока Белый дом транслирует прямую трансляцию.
  • Женщины в STEM: В сотрудничестве с Советом Белого дома по делам женщин и девочек Управление по политике в области науки и технологий (OSTP) инициировало ряд усилий по увеличению участия девочек в предметах STEM.

Филантропические инициативы STEM

  • Измените уравнение: эта некоммерческая организация, возглавляемая генеральными директорами, стремится мобилизовать бизнес-сообщество для повышения качества образования в области STEM в США.С.
  • Connect a Million Minds (CAMM): CAMM, спонсируемая Time Warner Cable, представляет собой пятилетнюю благотворительную инициативу стоимостью 100 миллионов долларов, которая направлена ​​на то, чтобы вдохновить студентов на развитие навыков STEM.
  • US2020.org: Конечной целью этой некоммерческой организации является мобилизация одного миллиона наставников STEM ежегодно к 2020 году.
  • Youth Inspired Challenge (YIC): созданный Ассоциацией научно-технологических центров (ASTC), YIC призван расширить влияние обучения STEM за пределами классной комнаты.

STEM Fun для младших школьников ↑

Классные веб-сайты STEM

  • Funology: В Funology наука обязательно должна стать интерактивной.Сделайте смерч из воды. Постройте террариум в Парке Юрского периода. Или просто мучайте своих братьев и сестер бесконечными шутками о жуках и насекомых.
  • Помощь вашему ребенку в изучении математики: это может заинтересовать ваших родителей. Этот веб-сайт, курируемый Министерством образования США, содержит задания по математике (выполняемые дома, в магазине и в дороге) для дошкольников и младших школьников.
  • Kids Do Ecology: Каждый ребенок должен быть экологическим героем. Узнайте о биомах, синих китах и ​​сборе данных.Вы даже можете создать свой собственный эксперимент в классе. Доступно на испанском языке.
  • Kids.gov: от воображаемых джунглей до ионных экспериментов на Kids.gov есть множество ресурсов для дождливого дня. Посмотрите анимацию о громе и молнии или совершите виртуальную экскурсию в Национальный зоопарк.
  • The Kids ’Science Challenge (KSC): практические научные занятия, игры, интересные видео, охота за мусорщиками… этот веб-сайт полон забавных вещей. KSC также проводит бесплатные общенациональные соревнования по естествознанию для учащихся 3–6 классов.
  • Детский клуб НАСА: в детском клубе НАСА совершенно нормально дурачиться в космосе. Вы можете использовать свои естественные науки и математические навыки, чтобы исследовать Марс, построить флот из ракет или искать побочные продукты НАСА в своем гараже.
  • NASA Space Place: создайте свой собственный космический корабль, поиграйте в космические вулканы или просмотрите галерею изображений солнца. Когда вы находитесь в Space Place, вселенная — это предел.
  • National Geographic Kids: Что вам симпатичнее: рыба фугу или рыба-клоун? На этом сайте вы можете проголосовать в опросах, принять участие в яиц-периментах, посмотреть видео, сыграть в головоломки и узнать удивительные факты.
  • Weather Wiz Kids: познакомьтесь с метеорологом Кристал Уикер. Она создала веб-сайт, на котором все объясняется о погоде. Найдите забавные факты, игры, карточки и фотографии, а также получите ответы на свои метеорологические вопросы.
  • Virtual Tech Camps: круглогодичное онлайн-обучение для детей и подростков с учебной программой на Python, Java, Minecraft, Roblox, AI, Unreal Engine, Adobe, 3D-моделировании и многом другом. Сэкономьте 100 долларов с кодом ВМЕСТЕ.
  • Кибер-академия Карнеги: В кибер-академии Карнеги есть интерактивные игры, которые учит детей правилам безопасности при работе в Интернете.
  • В
  • Wonderville: Wondervlile есть все — научные эксперименты, комиксы, видео, мероприятия на такие темы, как солнечная энергия и пожаротушение с воздуха.
  • Учебный центр Kinder Care: Учебный центр Kinder Care ориентирован на обеспечение непрерывного образования этим любознательным людям во время зимних и летних каникул.

PBS Дети

  • Cyberchase: Помогите Джеки, Мэтту и Инес использовать математику, чтобы защитить цифровую вселенную от зла. Не волнуйтесь: в Cyberchase есть множество математических игр, видео и заданий, которые помогут вам в ваших поисках.
  • Design Squad Nation: Создавайте все, что (!) Вы можете себе представить. С помощью заданий Design Squad, видео и руководств вы узнаете все, что нужно знать о принципах проектирования.
  • Кот в шляпе знает об этом толк !: игр, заданий и видеороликов Pre-K STEM. Авантюрный Кот в шляпе готов повести вас в экзотическое математическое сафари-приключение
  • Спасательная шлюпка на Марс: исследуйте мир биологии в этой бесплатной онлайн-игре. В одном симуляторе (Microland) вы управляете голодными микробами.В другом (Ecoland) вам нужно сбалансировать экосистему космической станции.
  • Zoom: горячая наука и крутые идеи. На веб-сайте Zoom вы найдете всевозможные занятия и эксперименты, в том числе ракеты с лимонным соком, сумасшедшие соломенные мосты и города-пузыри.
  • Design Squad Global: Хотите участвовать в интересных научных задачах, играть в игры и смотреть забавные видео? Ознакомьтесь с Design Squad Global.

Научные игры и приложения

  • Amazing Alex App: В Amazing Alex есть множество безумных физических задач, требующих ваших изобретательных решений.Вы даже можете строить и создавать свои собственные. Принесено вам создателями Angry Birds.
  • Angry Birds Space App: эти дурацкие (и безумно успешные) птички теперь разыгрывают свои физические головоломки в космосе, где гравитация творит довольно странные вещи!
  • У каждого тела есть мозг!: Погрузитесь в свой удивительный мозг с помощью песен, анимации и мини-игр. Полную версию игры можно приобрести на компакт-диске или загрузить в цифровом виде.
  • Geo Walk: 3D World Factbook App: радуйтесь географии! Это обучающее приложение содержит изображения и факты о сотнях мест, растений и животных.
  • Kinectic City: удивительная коллекция научных экспериментов, игр, заданий и задач. Вы можете запустить эстафету клеток крови или использовать компьютерную модель, чтобы построить свой собственный бизнес в межзвездной слякоти.
  • Max и приложение Magic Marker: в этой увлекательной игре, основанной на физике, вы полностью контролируете Макса и его невероятный магический маркер. Есть 15 уровней головоломок, на каждом из которых есть задачи, секреты и награды.
  • Move the Turtle: Programming for Kids: не нужно быть компьютерным гением, чтобы программировать! С помощью этого приложения любой ребенок может изучить азы программирования в графической среде.
  • сезонов! Приложение: куда бы вы ни пошли, всегда берите с собой погоду. В этом приложении вы узнаете, как определять различные погодные условия в разное время года. Для детей от 3 до 6 лет.
  • Приложение Sid’s Science Fair: у Сида из PBS «Sid the Science Kid» есть три научные игры для вашего развлечения: «Collection Inspection» Габриэлы, «Chart It!» Мэй. и «Машина времени» Джеральда. Для детей от 3 до 6 лет.
  • Команда Умизуми: веселые анимированные персонажи из Ника-младшего.Телепрограмма предлагает множество математических игр и заданий для дошкольников.
  • Чикагский музей науки и промышленности — эксперимент: познавайте и изучайте науку с помощью интерактивных мобильных приложений и выполняйте практические задания, которые вы можете попробовать дома.

Математические игры и приложения

  • Приложение Geometry Quest: путешествуйте по миру, решая геометрические задачи на своем пути. За совершенные квесты вы получите штампы в паспорте. Охватывает стандарты Common Core 3MD, 3G, 4MD, 5G, 6G, 7G и 8G.
  • Math Blaster: Сможете ли вы спасти галактику? Вам понадобятся математические навыки для выполнения тренировочных заданий в этой бесплатной онлайн-игре.
  • MathBoard App: одно для родителей. Это полезное приложение знакомит детей с этапами решения уравнений сложения, вычитания, умножения и деления. Есть удобная доска для рисования, где дети могут решать задачи вручную.
  • Математика движения: Пицца! Приложение: Пицца, пицца! В этой математической игре вы покупаете ингредиенты, разрабатываете фирменные пиццы и продаете их клиентам (надеюсь, с прибылью).
  • Motion Math: Questimate! Приложение: насколько быстро самый быстрый поезд в мире? Сколько мармеладов заполняет футбольный мяч? В Questimate !, вы можете придумывать свои собственные вопросы.
  • Mystery Math Town: Ваша миссия, если вы решите принять ее, — спасти светлячков, спрятанных в Mystery Math Town. Будьте осторожны: вам понадобятся математические навыки, чтобы открыть все комнаты и проходы в вашем квесте!
  • Числовая лига: в Числовой лиге только математика может спасти положение. Вы будете использовать все, от сложения до отрицательных чисел, чтобы собрать команду супергероев и поймать орду злодеев.
  • Умиго: Все надоело? У сумасшедших персонажей UMIGO есть ответ. Их интерактивные игры как раз подходят для развития математических навыков и навыков критического мышления.
  • Brain Pop Jr .: Вы любите математику? Хотите быстро вычислять числа? В Brain Pop есть все советы и хитрости, чтобы научиться лучше разбираться в числах.
  • CoolMath5Kids: Cool4Math — отличный ресурс для детей, которые любят играть в игры. Он сочетает в себе обучение с играми, что доставляет дополнительные математические удовольствия.
  • Fun Brain: Funbrain сортируется по типу класса, он предназначен для детей от дошкольного до восьмого классов. В нем есть все необходимые математические ресурсы для ваших детей.

Конкурсы STEM

  • Junior FIRST® LEGO® League: вы фанат LEGO®? Тогда это конкурс для вас. Вы будете использовать кубики LEGO®, чтобы спроектировать и построить движущуюся модель; Затем вы соберете плакат Show Me, чтобы продемонстрировать свое решение. Для детей от 6 до 9 лет.
  • NSBE KidZone Elementary Science Olympiad: соберите команду и проверьте свои научные навыки в 18 различных мероприятиях на национальном съезде Национального общества чернокожих инженеров (NSBE).Открыт для учащихся с третьего по пятый класс. Учащиеся от детского сада до второго класса соревнуются в неконкурентной лиге.
  • Многолетние математические турниры: виртуальный математический турнир (посредством видеоконференцсвязи) для команд и отдельных лиц. Открыт для учащихся с третьего по восьмой класс.

Лагеря STEM

  • Природные лагеря Одубона: Аудобон предлагает множество природных лагерей по всей стране. С апреля они начинают принимать заявки на участие в программе «Пути диких птиц к природе».
  • Изобретение лагеря: Мечты становятся открытием в этом летнем дневном лагере.Созданный Национальным залом славы изобретателей, Camp Invention представляет основные концепции STEM посредством творческих практических занятий.
  • Cosmophere Camps: В бесконечность и дальше! Эти крутые тематические лагеря, посвященные полетам и космическим приключениям, проходят в связанном со Смитсоновским институтом Канзасском космическом и космическом центре (KAOS) в Хатчинсоне, штат Канзас.
  • Destination Science Camp: проведите этим летом неделю, создавая роботов, создавая цифровую музыкальную систему, тренируя хамелеона с электроприводом или даже готовясь к миссии на Луну! Проводится в 130 точках в шести штатах.
  • Digital Media Academy Adventures Camp: отмеченные наградами лагеря Digital Media охватывают все: от создания мультфильмов до компьютерного программирования и продвинутой робототехники с LEGO® EV3. Для детей от 8 до 12 лет.
  • Engineering for Kids: Engineering for Kids — образовательная компания для детей от 4 до 14 лет. Она предлагает множество программ STEM, в том числе школьные экскурсии, дни рождения, семинары и лагеря.
  • лагерей iD Tech: без ограничений в дневных и ночных лагерях iD Tech.Создайте свою собственную видеоигру, запрограммируйте собственное приложение или даже код на Java.
  • Летние лагеря KinderCare®: от дурацкой науки о воде до основ выживания в дикой природе — KinderCare предлагает множество программ для детей от дошкольного до школьного возраста.
  • Исследователи науки: акулы и подводные лодки, зелья и слизь, замки и катапульты … что бы вы ни любили, в этих летних научных лагерях найдется занятие для вас. Предлагается в Пенсильвании, Нью-Джерси и Делавэре.
  • Snapology: Snapology сотрудничает со школами по всей стране, предлагая программы, конкурсы и лагеря STEAM.Программы являются интерактивными, что позволяет детям учиться с помощью практических инструкций и игр. Программы предлагаются в различных форматах, в том числе после уроков, по выходным и летом.
  • Vision Tech Camps: Vision Tech предлагает лагеря для детей от 7 до 17 лет в районе залива Сан-Франциско. Темы лагеря включают робототехнику, программирование, майнкрафт и многое другое.
  • Молодежные цифровые летние лагеря: создавайте трехмерные модели для Minecraft, создавайте свои собственные видеоигры или даже создавайте трехмерную анимацию! Эти лагеря, ориентированные на цифровые технологии, проходят в разных южных городах.Для детей от 8 до 16 лет.

Карьерные ресурсы STEM

  • Карьера Проходка: Элементарно: Мечтаете о том, кем хотите стать, когда вырастете? Эти видеоролики о вакансиях в области науки, технологий, инженерии и математики могут помочь вам принять решение.

Примечание. Многие государственные и региональные организации не попали в этот список. Если вас интересуют местные лагеря, стипендии и внеклассные мероприятия, обратитесь к учителям и в школе.

STEM Fun для детей средней школы ↑

Классные веб-сайты STEM

  • Теория большого мозга — Discovery Channel: У участников этого телешоу есть всего 30 минут, чтобы придумать решение (казалось бы) невозможной инженерной задачи.
  • Билл Най, ученый. В развлекательных телесериалах Билла рассказывается обо всем, от комет до музыки. Повеселитесь с его домашними демо.
  • Чи Альфа Му: иначе известный как Национальный клуб младших математиков, Чи Альфа Му является младшим братом Му Альфа Тета. Ознакомьтесь со списком конкурсов и летних грантов.
  • Студенческий портал по гигиене окружающей среды: Хотите узнать больше о химических веществах, качестве воздуха и загрязнении воды? На этом веб-сайте есть видео, игры и эксперименты, которые помогут вам в этом.
  • Дети впереди: золотое дно STEM. Kids Ahead наполнен всевозможными ресурсами, включая поиски мусора, видео, статьи, ссылки на местные мероприятия и веселые события, а также информацию о крутых вакансиях, которые вдохновляют и волнуют.
  • MathMovesU: Оттачивайте свои математические навыки с помощью онлайн-игр, виртуальных аттракционов и национальных соревнований! MathMovesU также предлагает различные стипендии и спонсорство.
  • MythBusters — Discovery Channel: сотрудники MythBusters используют эксперименты, чтобы опровергнуть слухи, мифы и городские легенды.(Во время эксперимента по химии пушечного ядра они случайно вонзили пушечное ядро ​​в стену дома.)
  • Sally Ride Science: основанная первой в Америке женщиной-астронавтом, Sally Ride Science проводит ряд студенческих программ, включая научные фестивали и ночевки.
  • Наука Боб: Боб — учитель естественных наук, который любит экспериментировать (часто с Джимми Киммелом). На его веб-сайте есть видео, ссылки и множество идей для собственных экспериментов и проектов для научных ярмарок.
  • SciJinks: Все дело в погоде. Национальная ассоциация океанических и атмосферных исследований (NOAA) и НАСА создали этот образовательный веб-сайт, чтобы обучать детей метеорологии и наукам о Земле. Загляните в их раздел игр.
  • Scratch: Созданный для детей от 8 до 16 лет, Scratch — это место, где вы можете программировать свои собственные интерактивные истории, игры и анимации. Проект группы Lifelong Kindergarten Group в MIT Media Lab.
  • Virtual Tech Camps: круглогодичное онлайн-обучение для детей и подростков с учебной программой на Python, Java, Minecraft, Roblox, AI, Unreal Engine, Adobe, 3D-моделировании и многом другом.Сэкономьте 100 долларов с кодом ВМЕСТЕ.

STEM-игры и приложения

  • Auditorium: The Online Experience: Auditorium — это красивая и сложная головоломка с множеством различных решений. Один обозреватель игры назвал это «частично головоломкой, частично световой скульптурой, частично музыкальным инструментом».
  • CSI: Web Adventures: это захватывающее приключение, основанное на сериале T.V., позволяет вам раскрыть собственное судебное дело. Уровни варьируются от начального до продвинутого.
  • DimensionU Games: DimensionU разработала множество игр, в которых используются STEM-навыки.Используйте математику, чтобы раскрыть тайны острова Ксено, или объедините силы в гонке, чтобы избавиться от биоцифрового вируса.
  • Gamestar Mechanic: научитесь создавать свою собственную видеоигру! Исследуйте игровые квесты и пройдите курсы, чтобы развить свои навыки.
  • Machinarium: невероятно увлекательная приключенческая игра в жанре «укажи и щелкни». Вы — робот, брошенный на свалку, и вам нужно решить ряд головоломок, чтобы вернуться в город, спасти девушку и победить плохих парней.
  • Mathemagics Mental Math Tricks: поразите друзей и родителей этими быстрыми (но впечатляющими) математическими уловками.
  • Minecraft: Minecraft — популярная игра с трехмерным построением блоков, которая доводит ваше воображение до предела. Защитите себя от ночных монстров или постройте гигантское единственное в своем роде создание.
  • National Geographic Games: Путешествуйте вглубь нано-мира. Постройте самый зеленый город во вселенной. Готовьтесь к апокалипсису. Некоторые из этих игр бесплатны; некоторые должны быть куплены.
  • Portal 2: умопомрачительная приключенческая игра, построенная на принципах физики и головоломках, связанных с окружающей средой.Перемещайтесь по порталам и сражайтесь с помешанным на мощи искусственным интеллектом GLaDOS. Подходит для подростков.
  • Квантовая головоломка: исчез ваш дядя. Он оставил свое устройство межпространственного сдвига. А его дом стал очень странным. Добро пожаловать в основанную на физике головоломку, известную как Quantum Conundrum.
  • Роботы для iPad: все, что вы хотите знать о роботах, в одном удобном приложении. Роботы для iPad имеют обзоры на 360 градусов, множество статей и спецификаций, а также сотни фотографий и видео.
  • Кубик Рубика можно сделать: вы знали, что должна быть игра, полностью посвященная ему. Раскройте секреты всемирно известного кубика Рубика.

Лагеря STEM

  • Программа амбиций: смело отправляйтесь туда, где раньше не ходил ни один ребенок. Погрузитесь в шестидневное учебное приключение на авиационную тематику в Национальной летной академии во Флориде.
  • Природные лагеря Одубона: в Аудобоне расположено огромное количество природных лагерей по всей стране.
  • Лагерь Евклида: Математический исследовательский лагерь: участвуйте практически отовсюду! Шестинедельные летние лагеря Camp Euclid проводятся онлайн.Сотрудничайте с сокурсниками по математическим задачам, не требующим решения.
  • Cosmophere Camps: Откройте для себя острые ощущения космоса. Эти захватывающие тематические лагеря, посвященные полетам и космическим приключениям, проходят в принадлежащем Смитсоновскому институту Канзасском космическом и космическом центре (KAOS) в Хатчинсоне, штат Канзас.
  • Летний лагерь Digital Media для подростков: отмеченные наградами летние лагеря Digital Media предназначены для подростков от 12 до 17 лет. Узнайте о дизайне и разработке игр, программировании и приложениях, кинопроизводстве и визуальных эффектах или трехмерном моделировании и анимации.
  • Earth Camp: исследуйте чудеса пустыни Сонора в Аризоне. Вы разместитесь лагерем в дикой местности, исследуете ночное небо в Sky Center Университета Аризоны и станете экспертом в вопросах устойчивого развития и водных ресурсов.
  • Летние лагеря инженеров: Хотите решить проблемы этим летом? Сервисный центр инженерного образования составил штатный перечень летних лагерей инженеров.
  • Летний научный лагерь Бернарда Харриса ExxonMobil (EMBHSSC): живите (и играйте) в реальном кампусе колледжа.Эти популярные летние научные лагеря, созданные для поддержки недостаточно представленных учеников средней школы, расположены по всей стране.
  • iD Tech Camps: создайте свою собственную видеоигру. Запрограммируйте собственное приложение. Код на Java. В дневных и ночевых лагерях iD Tech возможно практически все.
  • Лагеря STEM Университета Северного Иллинойса: Университет Северного Иллинойса проводит летние лагеря STEM, которые позволяют учащимся средних школ заниматься междисциплинарной деятельностью. Студенты учатся на занятиях, на практических занятиях и во многом другом!
  • Летний институт STEM при Массачусетском технологическом институте: Летом STEM предлагает пятинедельный математический и научный институт при Массачусетском технологическом институте для учащихся, поступающих в классы с шестого по девятый.В стоимость включены экскурсии и занятия ракетным спортом.
  • Vision Tech Camps: Vision Tech предлагает лагеря для детей от 7 до 17 лет в районе Сан-Франциско. Лагеря посвящены таким темам, как инженерия, игровой дизайн, робототехника и многое другое.
  • Молодежные цифровые летние лагеря: создавайте трехмерные модели для Minecraft, создавайте свои собственные видеоигры или даже создавайте трехмерную анимацию! Эти лагеря, ориентированные на цифровые технологии, проходят в разных южных городах. Для детей от 8 до 16 лет.
  • Youth Empowered Action (YEA): YEA — это недельный лагерь с ночёвкой для молодежи в возрасте от 12 до 17 лет, которая хочет изменить мир.Семинары включают «Головоломки планетарных проблем» и «Миллион способов изменить мир к лучшему».
  • Летняя программа средней школы Zero Robotics: начните изучать компьютерное программирование, робототехнику и космическую инженерию. Пятинедельная учебная программа MIT по STEM погрузит вас в пространство и предоставит вам практический опыт программирования СФЕР (синхронизация, положение, удержание, взаимодействие, переориентация, экспериментальные спутники).

Конкурсы науки и техники

  • Discovery Channel Young Scientist Challenge: в этом единственном в своем роде конкурсе вам будет предложено создать одно-двухминутное видео, описывающее новое и инновационное решение, которое может решить повседневную проблему.Главный приз — 25000 долларов и международная поездка!
  • ЭКИБЕРМИССИЯ: Выполняя миссию (например, альтернативные источники энергии) с вашей командой, у вас есть шанс выиграть 5000 долларов в виде сберегательных облигаций и грант STEM-in-Action, чтобы ваше решение работало в вашем сообществе.
  • FIRST® LEGO® League: спроектируйте, соберите и запрограммируйте своего собственного робота с помощью технологии LEGO MINDSTORMS®! Набирайте очки на тематическом игровом поле и разрабатывайте решения реальных задач.
  • Конкурс городов будущего: если вы можете себе это представить, вы можете построить.Работая с педагогом и наставником инженеров, вы будете планировать город с помощью программного обеспечения SimCity ™, искать решения инженерных проблем и создавать настольные модели из переработанных материалов.
  • Национальная лига
  • (NSL): ранее известная как Ten80 Student Racing Challenge, NSL предлагает четыре различных конкурса для учащихся средних и старших классов. Вы можете спроектировать быстрый, эффективный и стабильный гоночный автомобиль в Racing Challenge, научить робота проходить курс в Rover Challenge, перейти на возобновляемые источники энергии в Energy Challenge или сделать что-то совершенно новое в Innovation Challenge.
  • National STEM Video Game Challenge: представьте свой оригинальный игровой дизайн, созданный с помощью таких инструментов, как Gamestar Mechanic, Scratch и Kodu. Победитель получает портативный компьютер на базе процессора AMD с программным обеспечением для игрового дизайна и 2000 долларов для своей школы.
  • NSBE Jr. Bridge Magazine Contests: Национальное общество чернокожих инженеров спонсирует различные конкурсы, в которых вам предлагается продемонстрировать свои навыки STEM или повысить осведомленность о проблемах в STEM.
  • Конкурс технических инноваций NSBE Jr. Explorer: сразитесь с другими студентами-учеными на ежегодном съезде NSBE.Учащиеся средних и старших классов имеют право на участие. Для участия вы должны быть платным участником NSBE Jr.

Соревнования по математике

  • AMC 8: Проверьте свои математические навыки в этом 40-минутном конкурсе с несколькими вариантами ответов из 25 вопросов (проводится каждый ноябрь).
  • Соревнования MATHCOUNTS: MATHCOUNTS проводит серию конкурсов «пчелиный стиль» в более чем 500 местных отделениях. Лучшие команды проходят на государственные соревнования, а затем в Национальные соревнования в мае.
  • MATHCOUNTS Math Video Challenge: создайте собственное математическое видео с друзьями и одноклассниками и участвуйте в гонках, чтобы выиграть стипендию в колледже!
  • Многолетние математические турниры: виртуальный математический турнир (посредством видеоконференцсвязи) для команд и отдельных лиц.Открыт для учащихся с третьего по восьмой класс.
  • Rocket City Math League (RCML): спонсируемый Mu Alpha Theta, RCML — это ежегодное соревнование по математике, состоящее из четырех раундов. В конце года призы рассылаются по почте лучшим ученикам средних и старших классов.
  • USA Mathematical Talent Search (USAMTS): сравните свои навыки решения проблем с одними из самых сложных головоломок. Из-за уровня сложности USAMTS дает студентам целый месяц или более для выработки решений.

Карьерные ресурсы STEM

  • Карьерные исследования !: STEM: Хотите знать, что нас ждет в будущем? Изучите некоторые из доступных вам вариантов в области естественных наук, технологий, инженерии и математики.Много видео.
  • Kids.gov Работа: получайте худощавую на любой работе под солнцем. Хотите знать, чем занимаются морские биологи? Хотите посмотреть видео о том, как стать ветеринаром? Вы попали в нужное место.
  • НАСА: взгляд в будущее: карьера в космосе: вам не нужно быть астронавтом, чтобы работать в космической программе. У НАСА есть список других профессий, в том числе инженер-робототехник, ученый-компьютерщик и океанограф, на ваш выбор.

Примечание. Многие государственные и региональные организации не попали в этот список.Если вас интересуют местные лагеря, стипендии и внеклассные мероприятия, обратитесь к учителям и в школе.

STEM Fun для старшеклассников ↑

Классные веб-сайты STEM

  • Arrick Robotics: это самый красивый веб-сайт в мире, но если вы ищете ресурсы по робототехнике, вам сюда. Включает списки соревнований и конкурсов, групп и клубов, игр и симуляторов.
  • Codeacademy: научитесь программировать в интерактивном режиме (и бесплатно).Codeacademy предлагает классы кодирования на основных языках программирования, таких как Python, PHP, jQuery, JavaScript и Ruby.
  • DiscoverE: Думаете о технике? DiscoverE предлагает подборку ресурсов о карьере, подготовке к поступлению в колледж и исследовательских школах. Вы также можете проверить их список видео, поездок, веб-сайтов и практических занятий.
  • Му Альфа Тета: Му Альфа Тета, также известная как Национальное математическое общество средней школы и двухлетнего колледжа, насчитывает более 100 000 учеников.Он организует национальный математический конгресс, предлагает специальные награды и проводит конкурсы.
  • Студенческая наука: центральное место для научных новостей, блогов, ресурсов и информации о соревнованиях Intel. Примеры названий статей включают «Родные сопли» и «Библиотека без книг».
  • Virtual Tech Camps: круглогодичное онлайн-обучение для детей и подростков с учебной программой на Python, Java, Minecraft, Roblox, AI, Unreal Engine, Adobe, 3D-моделировании и многом другом. Сэкономьте 100 долларов с кодом ВМЕСТЕ.

STEM-игры и приложения

  • Приложение Algebra Touch: освежите свои навыки алгебры с помощью этого сенсорного инструмента. Нажмите, чтобы упростить, перетащите, чтобы изменить порядок, и нарисуйте линии, чтобы исключить идентичные термины.
  • The Elements: Если вы так же увлекаетесь периодической таблицей Менделеева, как и я, вам понадобится это приложение. Узнайте текущую цену на золото, узнайте период полураспада плутония или прочитайте информацию о гелий-неоновых лазерах.
  • Межпланетное приложение «3D Солнце»: этот инструмент, спонсируемый НАСА, собирает данные с флота космических аппаратов НАСА.Наблюдайте за солнечными вспышками, выбросами корональной массы и геомагнитными бурями сразу после их возникновения.
  • Приложение
  • Muscle System Pro III: снимите плоть, чтобы узнать, что находится под ней. Это интерактивное приложение, разработанное в сотрудничестве с Медицинской школой Стэнфордского университета, позволяет исследовать работу мускулатуры человека слой за слоем.
  • Приложение
  • НАСА: обязательное условие для поклонников НАСА. Это приложение-монстр включает в себя прямую трансляцию телеканалов НАСА и более 13 000 изображений, а также видео по запросу, новости и возможности для наблюдения за Международной космической станцией (МКС).Это тоже бывает бесплатно.
  • приложений National Geographic. В National Geographic есть чем развлечься в унылый день. Среди приложений с самым высоким рейтингом — Национальные парки и Мировой Атлас.
  • Pocket Universe App: безграничная астрономия. Совершите виртуальный визит на поверхность Марса. Оживите ночное небо. Играйте в викторины. Получайте всплывающие уведомления об астрономических событиях.
  • Virtual Frog Dissection: Все образование без кишки. Это приложение позволяет вам использовать инструменты виртуального препарирования, чтобы раскрыть тайны анатомии земноводных.

Лагеря STEM

  • Летняя исследовательская академия на Аляске (ASRA) — средняя школа: в ASRA вы проведете две недели в кампусе Университета Аляски в Фэрбенксе, работая в небольших группах и участвуя в проектном обучении. Некоторые модули доставят вас в отдаленные районы Аляски для полевых исследований.
  • Программа амбиций: приготовьтесь к захватывающей поездке. На шесть дней вы погрузитесь в учебное приключение на авиационную тематику в Национальной летной академии в Пенсаколе, штат Флорида.
  • Природные лагеря Одубона: в Аудобоне расположено огромное количество природных лагерей по всей стране.
  • Camp Euclid: A Mathematics Research Camp: Шестинедельные виртуальные летние лагеря Camp Euclid проводятся онлайн. Совместно с однокурсниками решайте невероятно сложные математические задачи.
  • Cosmosphere Camps: Эти тематические лагеря, посвященные полетам и космическим приключениям, проходят в Канзасском космическом и космическом центре (KAOS), входящем в состав Смитсоновского института, в Хатчинсоне, штат Канзас.
  • Летний лагерь по цифровым медиа для подростков: приступайте к творчеству с дизайном и разработкой игр, программированием и приложениями, созданием фильмов и визуальными эффектами или трехмерным моделированием и анимацией.Отмеченные наградами летние лагеря Digital Media предназначены для подростков от 12 до 17 лет.
  • Earth Camp: исследуйте глобальные изменения климата, воды и ландшафта, сплавляясь по каньону Пустоши Грин-Ривер в Центральной Юте. Управляется Научным колледжем Университета Аризоны, проектом WET, Институтом планетологии и Музеем пустыни Аризоны.
  • Летние лагеря инженеров: ознакомьтесь с этим списком инженерных лагерей по штатам для ближайшего к вам летнего лагеря.
  • iD Game Design & Development Academy: эти двухнедельные летние лагеря предлагают интенсивное погружение в разработку игр, программирование, дизайн, трехмерное моделирование и анимацию.Выбирайте из курсов по Minecraft, Unreal® Engine, Maya®, iPhone® и другим. Для подростков от 13 до 18 лет.
  • Game Camp Nation: Game Camp Nation предлагает развлекательные программы, которые раскрывают страсть вашего ребенка к видеоиграм. У них есть места на восточном побережье от Массачусетса до Атланты для детей от 7 до 16 лет. В настоящее время они предлагают несколько программ, включая Game Design с Tynker, Coding & Minecraft Modding с Java и Программирование 3D-игр с Unity.
  • Академия программирования
  • iD: идеально подходит для студентов с предыдущим опытом программирования, которые хотят вывести свои навыки программирования на новый уровень.Лагеря проводятся в университетских городках США для подростков от 13 до 18 лет.
  • iD Tech Camps для подростков: выберите свое собственное приключение. Недельные летние лагеря iD позволяют вам запрограммировать новое приложение, снять фильм, разработать веб-сайт — практически все, что связано с технологиями. Для подростков от 13 до 17 лет.
  • STEM-лагеря Университета Северного Иллинойса: NIU предлагает несколько летних STEM-лагерей для старшеклассников, в том числе STEM Career Explorations, Crisis on Mars! И STEAM Camp Eagle’s Nest.
  • Программа стажировки в области естественных наук и инженерии (SEAP): интересуетесь естественными науками или математикой? Затем вы могли бы пройти стажировку в течение восьми недель в лаборатории Министерства военно-морского флота (DoN).В большинстве лабораторий требуется, чтобы учащиеся были не моложе 16 лет (хотя иногда допускаются и 15-летние).
  • Summer Academy for Mathematics and Science (SAMS): Конкурсная летняя программа Carnegie Mellon предназначена для многообещающих студентов, поступающих в младшие или старшие классы средней школы и планирующих карьеру в STEM. Нагрузка на курс довольно высока, но если вас выбрали, плата за обучение, проживание или питание не взимается.
  • Vision Tech Camps: Vision Tech предлагает летние лагеря для детей в возрасте от 7 до 17 лет в районе залива Сан-Франциско.Дети пройдут курсы инженерии, робототехники, программирования, игрового дизайна и других интересных технических тем.
  • Молодежные цифровые летние лагеря: создавайте трехмерные модели для Minecraft, создавайте свои собственные видеоигры или даже создавайте трехмерную анимацию! Эти лагеря, посвященные цифровым технологиям, проходят в различных южных городах. Для детей от 8 до 16 лет.
  • Youth Empowered Action (YEA): YEA — это недельный лагерь с ночёвкой для детей от 12 до 17 лет, которые хотят изменить мир. Семинары включают «Головоломки планетарных проблем» и «Миллион способов изменить мир к лучшему.”

Конкурсы науки и техники

  • AbilityOne Design Challenge: задача с определенной целью. Вы будете исследовать, разрабатывать и разрабатывать технологии, которые позволят людям с ограниченными возможностями найти новую работу или повысить производительность труда на рабочем месте.
  • Air Force Association (AFA) CyberPatriot Competition: устранение реальных ситуаций кибербезопасности в виртуальной среде. Ранние туры проходят онлайн в выходные осенью, зимой и весной; лучшие команды приглашены в Вашингтон, Д.C. принять участие в Национальном финале конкурса.
  • Образовательный центр математики и информатики: CEMC Университета Ватерлоо содержит всемирно признанные материалы, призванные помочь детям влюбиться в математику и информатику.
  • Envirothon: соревнуйтесь за награды и стипендии, демонстрируя свои знания в области наук об окружающей среде и управления природными ресурсами. Команды проходят через местные соревнования Envirothon в недельные летние финалы в июле или августе.
  • FIRST® Robotics Competition (FRC): создавайте, программируйте и соревнуйтесь с роботом собственной конструкции. Изучите сложное оборудование, работайте с профессиональными инженерами и получите право на студенческие стипендии.
  • FIRST® Tech Challenge (FTC): Близкий родственник FRC, FTC предлагает вам создать робота, которого вы можете использовать, чтобы соревноваться в формате альянса с другими командами. Вы получите практический опыт программирования и быстрого прототипирования.
  • Intel International Science and Engineering Fair (ISEF): Годзилла научных ярмарок.Около 1800 новаторов приглашаются принять участие в недельном празднике науки, технологий, инженерии и математики. Разыграны премии и стипендии на сумму более 5 миллионов долларов.
  • Intel Science Talent Search (STS): Intel STS позиционирует себя как старейшее и самое престижное национальное соревнование по науке перед колледжем. Сорок финалистов соревнуются за награды в размере 630 000 долларов и приз за первое место в размере 100 000 долларов. Это грандиозно: восемь выпускников получили Нобелевскую премию.
  • Серия NASA Asteroid Grand Challenge: станьте настоящим охотником за астероидами.В серии задач топкодера вам предстоит разработать значительно улучшенный алгоритм идентификации астероидов на изображениях, полученных с наземных телескопов.
  • Эксперименты по исследованию капиллярных эффектов НАСА в жидкостях (CELERE): CELERE, разработанный НАСА и Портлендским государственным университетом (PSU), открыт для студенческих команд с 9 по 12 классы и для групп с 5 по 12 классов. эксперимент, проверяющий влияние микрогравитации на действие капилляров; PSU проводит испытания на своей башне Dryden Drop Tower.
  • НАСА «Падение в условиях микрогравитации» (DIME): DIME открыт для студенческих команд, заинтересованных в разработке и проведении научного эксперимента, который можно проводить в условиях микрогравитации. Финалисты едут в Исследовательский центр Гленна, чтобы провести свои эксперименты в башне НАСА.
  • Национальная лига
  • (NSL): ранее известная как Ten80 Student Racing Challenge, NSL предлагает четыре различных конкурса для учащихся средних и старших классов. Вы можете спроектировать быстрый, эффективный и стабильный гоночный автомобиль в Racing Challenge, научить робота проходить курс в Rover Challenge, перейти на возобновляемые источники энергии в Energy Challenge или сделать что-то совершенно новое в Innovation Challenge
  • NSBE Jr.Конкурс технических инноваций Explorer: сразитесь с другими студентами-учеными на Ежегодном съезде NSBE. Учащиеся средних и старших классов имеют право на участие. Для участия вы должны быть платным участником NSBE Jr.
  • Team America Rocketry Challenge (TARC): спроектируйте, соберите и запустите свою собственную ракету. Разработанный Ассоциацией аэрокосмической промышленности, это единственный конкурс STEM в аэрокосмической отрасли в стране. Студенты соревнуются в командах от трех до 10 человек; команда-победитель забрала домой 10 500 долларов в 2014 году.
  • Турнир средней школы Zero Robotics: решите проблему, интересующую DARPA, NASA и MIT. Если вы пройдете контролируемое моделирование до финала, вы увидите, как ваш код запускается на спутниках SPHERES на борту Международной космической станции с прямой трансляцией из космоса.

Соревнования по математике

  • Американский экзамен по математике (AIME). Абитуриенты с высокими баллами AMC 10 и AMC 12 (см. Ниже) могут быть приглашены на трехчасовой экзамен AMAA из 15 вопросов.Лучшие бомбардиры этого теста попадают в USAMO (см. Ниже).
  • AMC 10/12: Каждый год AMAA предлагает 75-минутные экзамены по математике средней школы с 25 вопросами и несколькими вариантами ответов. Это первый шаг на пути к Международной математической олимпиаде (см. Ниже).
  • Соревнование силы Американской региональной математической лиги (ARML): в командную работу? Соревнование ARML Power Contest предоставит вам и вашим товарищам два набора задач, один осенью и один в конце зимы, каждый из которых необходимо решить в течение 45 минут.Трофеи награждаются 10 лучших команд.
  • Международная математическая олимпиада (IMO): если вы прошли AMC 10/12, AIME, USAMO и Летнюю программу математической олимпиады (MOSP), вам будет предложено соревноваться за США со сверстниками из более 90 стран на этом двухдневном экзамене.
  • Moody’s Mega Math (M3) Challenge: во время M3 у вас и небольшой команды юниоров и / или пожилых людей есть 14 часов на решение открытой задачи прикладного математического моделирования, ориентированной на реальную проблему.Вы можете работать из любого места. Победителям вручаются стипендии.
  • NSBE Jr. Try-Math-A-Lon: Национальное общество чернокожих инженеров разработало этот конкурс для обучения школьников математике, естественным наукам и истории афроамериканцев на уровне SAT. Победители из числа местных и региональных жителей направляются на Национальный съезд NSBE.
  • Пурпурная комета! Math Meet: название банальное, но у конкурса сильная репутация. В этом бесплатном онлайн-соревновании по математике вашей команде будет предложено 25 задач, которые нужно решить за 90 минут.
  • Rocket City Math League (RCML): спонсируемый Mu Alpha Theta, RCML — это ежегодное соревнование по математике, состоящее из четырех раундов. В конце года призы рассылаются по почте лучшим ученикам средних и старших классов.
  • Юношеская математическая олимпиада в США (USAJMO): На этот двухдневный экзамен приглашаются только лучшие участники AIME / AMC 10. Это включает шесть вопросов и девять часов экзаменов для эссе / корректуры. Лучшие бомбардиры переходят в Летнюю программу математической олимпиады (MASP).
  • U.Математическая олимпиада в Южной Америке (USAMO): На этот двухдневный экзамен приглашаются только лучшие бомбардиры AIME / AMC 12. Это включает шесть вопросов и девять часов экзаменов для эссе / корректуры. Лучшие бомбардиры переходят на Летнюю программу математической олимпиады (MASP).
  • USA Mathematical Talent Search (USAMTS): сравните свои навыки решения проблем с одними из самых сложных головоломок. Из-за уровня сложности USAMTS дает студентам целый месяц или более для выработки решений.
  • Кто хочет стать математиком?: Сражайтесь за деньги и призы, отвечая на вопросы по математике с несколькими вариантами ответов.Квалификационные тесты сдаются онлайн; полуфиналы и финалы проходят на совместных встречах по математике.

Гранты и возможности STEM

  • InvenTeam: InventTeams состоят из студентов, преподавателей и наставников, которые получают гранты в размере до 10 000 долларов США на разработку технологических решений реальных проблем (вы можете выбрать свою проблему).
  • Студенческие гранты
  • Planet Connect: Вы любите защищать дикую природу и естественную среду обитания? Planet Connect предлагает старшеклассникам гранты в размере 1000 долларов на реализацию местных проектов и участие в стажировках по дикой природе или природным ресурсам.

Карьерные ресурсы STEM

  • Science Splash: High School: на сайте Career Aisle вы найдете множество исследовательских видео, а также ссылки на информацию о заработной плате и ресурсы по подготовке к карьере.
  • Career Cornerstone Center: Он не выиграет никаких призов за красоту, но Career Cornerstone Center предлагает множество полезных ресурсов о карьере в STEM. Изучите более 185 дипломных областей, окунитесь в интервью или узнайте больше об образовательных требованиях, типичных зарплатах и ​​сетях.
  • CareerOneStop: узнайте все, что вам нужно знать о карьере в STEM, включая типичные профессии, стажировки и варианты обучения. При поддержке Министерства труда, занятости и обучения США.
  • IEEE Try Computing: хороший ресурс, если вы только начинаете изучать вычисления. Вы можете изучить варианты карьеры и специальности, искать аккредитованные программы и поработать с визуальным облачным инструментом карьеры.
  • IEEE Try Engineering: этот веб-сайт включает поиск по университетам, информацию по инженерным специальностям и длинный список ссылок на лагеря, стажировки, стипендии, конкурсы и многое другое.Вы также найдете мнения экспертов и виртуальные инженерные игры.
  • IEEE Try Nano: IEEE работает. На третьем этапе своей карьеры (см. Выше) они рассматривают вакансии в области нанонауки и нанотехнологий: технические области, в которых основное внимание уделяется материи в наномасштабе.

Примечание. Многие государственные и региональные организации не попали в этот список. Если вас интересуют местные лагеря, стипендии и внеклассные мероприятия, обратитесь к учителям и в школе.

STEM Fun для девочек ↑

Классные веб-сайты STEM

  • CanTEEN: CanTEEN был разработан, чтобы помочь девушкам осваивать STEM-карьеру. Примите вызов (например, создайте свой собственный городской сад), играйте в такие игры, как «Click! Школа шпионажа »или узнайте больше о реальных образцах для подражания.
  • Engineer Girl !: Почему ты должен стать инженером? Позвольте этому сайту для девочек средней школы объяснить. Помимо интервью, викторин, интересных фактов и профилей, в нем есть ссылки на множество инженерных соревнований, клубов, программ и стипендий.
  • Для девушек в науке: будь тем, кем хочешь быть. Этот сайт, спонсируемый L’Oréal, предлагает всевозможные варианты STEM, включая видеоблог, профили женщин в науке, список летних лагерей и информацию о карьере.
  • Girls Communicating Care Connections (GC3): Хотите сделать карьеру в науке или технологиях? В этом молодежном сериале для девочек из незаслуженных групп есть много идей, которые стоит изучить.
  • Программа для девочек-скаутов STEM: расширяйте границы, делая мир лучше.Чтобы поддержать опыт STEM, девушки-скауты разработали три направления лидерства и несколько значков уровня владения STEM.
  • iWASwondering.org: этот веб-сайт, вдохновленный «Приключениями женщин в науке» и разработанный Национальной академией наук, предлагает вам изучить карьеры известных женщин-ученых.
  • PBS SciGirls: видео SciGirls — отличный ресурс для классной комнаты. В каждом эпизоде ​​рассказывается о разных группах девочек из средних школ, которые разрабатывают и создают проекты STEM.
  • Общество женщин-инженеров (SWE) K-12 Outreach: Стремитесь к успеху. На этом сайте вы найдете огромное количество инженерных ресурсов, включая ссылки на мероприятия, конкурсы, лагеря и стипендии.
  • Women @ NASA: Познакомьтесь с женщинами, которыми вы хотите быть. Этот сайт НАСА включает видеоинтервью и биографии сотрудников НАСА, а также информацию о карьере, мероприятиях и информационно-просветительских программах. Energy.gov имеет дочерний сайт под названием Women @ Energy.
  • Virtual Tech Camps: круглогодичное онлайн-обучение для детей и подростков с учебной программой на Python, Java, Minecraft, Roblox, AI, Unreal Engine, Adobe, 3D-моделировании и многом другом.Сэкономьте 100 долларов с кодом ВМЕСТЕ.

Награды STEM

  • Премия NSTA Angela: Национальная ассоциация учителей естественных наук присуждает сберегательную облигацию EE в размере 1000 долларов США одной ученице пятого-восьмого классов, которая занимается наукой или имеет прочные связи с ней.

Лагеря STEM

  • Camp Reach: Этот двухнедельный летний лагерь в Вустерском политехническом институте в Массачусетсе разработан для того, чтобы развить ваше инженерное воображение, от создания идеальной обуви до создания идеального мороженого.Для девочек, поступающих в седьмой класс.
  • Дизайн-Соединение-Создавайте! Физические лагеря для молодых женщин: живете в Северном Техасе или рядом с ним? Получите практическое введение в ключевые принципы AP Physics. Для старшеклассниц, поступающих в младшие классы.
  • DigiGirlz High Tech Camp: профессиональные лагеря Microsoft проводятся в США и за рубежом. У вас будет возможность послушать технических докладчиков, посетить экскурсии, пообщаться и получить практический опыт на семинарах. Вариативное расписание. Для старшеклассниц.
  • E2 @ UMD: Изучите инженерное дело в Университете Мэриленда. В течение одной летней недели вы будете участвовать в практических занятиях, лабораторных экспериментах, командных задачах и семинарах с профессиональными инженерами. Для начинающих юниоров и пожилых людей.
  • Girls ’Adventures in Mathematics, Engineering and Science (G.A.M.E.S.): станьте частью современной инженерной или научной лаборатории этим летом! В G.A.M.E.S. Университета Иллинойса вы будете работать над сложными проектами лагерей и встречаться с наставниками в технических областях.Для учеников с девяти до двенадцати классов.
  • девочек достигают успехов в спорте и физике (GRASP): пятидневный летний лагерь GRASP, организованный факультетом физики Университета штата Огайо, наполнен увлекательной физикой. Сотрудники и студенты ОГУ присутствуют на всех занятиях, чтобы поделиться своей любовью к этому предмету. Для девочек средней школы.
  • Girlstart: застрять на предметах STEM. Программы Girlstart в Остине (включая летние лагеря, субботние семинары STEM и Science Extravaganzas) открыты для девочек из детских садов до 16 лет.
  • студентов с потенциалом и интересом, рассматривающих инженерию (S.P.I.C.E.): Постройте новый мир. Благодаря мероприятиям, проектам, турам и беседам в Университете Мэриленда, Колледж-Парк, вы узнаете, как инженерные разработки используются для изменения облика планеты. Для девочек, поступающих в девятые и десятые классы.
  • Летняя научно-техническая программа Смита (SSEP): сто девочек, четыре недели, один невероятный опыт. В этом летнем лагере в Массачусетсе вы пройдете два увлекательных исследовательских курса.Для возрастающих с девяти до двенадцатиклассников.
  • Женщины в естествознании (WINS): эта внешкольная и летняя дополнительная программа по естественным наукам проводится Академией естественных наук Дрексельского университета бесплатно! Для перспективных восьмиклассников, планирующих посещать государственную или чартерную школу в Филадельфии.
  • Alexa Cafe: студенты работают небольшими сплоченными группами. Делая упор на предпринимательство, лидерство, идентичность бренда и благотворительность, вы будете развивать технические навыки в уникальной стильной обстановке вместе с технически подкованными женщинами-наставниками.Недельные дневные и ночные занятия по программированию, игровому дизайну, кинопроизводству и многому другому.

Примечание. Многие государственные и региональные организации не попали в этот список. Если вас интересуют местные лагеря, стипендии и внеклассные мероприятия, обратитесь к учителям и в школе.

заданий олимпиады по английскому языку | Битва при Геттисберге

Понимание чтения для 9

чт

Студенты класса

(34 задания) НАПРАВЛЕНИЯ:

В этом тесте вы прочитаете четыре текста.За каждым текстом следует 5

7 заданий. Вы должны выполнять задания после текста на основе того, что указано или подразумевается в этом тексте.

ТЕКСТ 1 Вопросы с 1 по 10

O

в листе для ответов укажите «

»

Т ’

если утверждение верно, ‘

F ’

если это ложь.

Активно-спортивные лагеря

Спортивные лагеря Action — это активный отдых для детей от пяти лет и взрослых.Мы предлагаем обучение более чем двадцати видам спорта в десяти различных центрах по всей Великобритании. Все центры открыты с апреля по октябрь, а некоторые открыты зимой для занятий по выходным. Предлагаемые виды спорта различаются от одного центра к другому, поэтому, если вы хотите заниматься чем-то конкретным, вам следует ознакомиться с нашей цветной брошюрой. Лагеря не ограничиваются только спортом на открытом воздухе — мы также предлагаем широкий спектр мероприятий в помещении. Так что, если пойдет дождь, сборы продолжатся, хотя вам, возможно, придется снять футбольные бутсы и вместо этого взять ракетку для сквоша.Опираясь на многолетний опыт, мы создали правильное сочетание спорта и занятий, обеспечивающих спорт для всех, а не только для тех, кто блестяще занимается легкой атлетикой. Нет необходимости приносить какое-либо оборудование, потому что оно все предусмотрено. Мы работаем в небольших группах, дети работают с другими людьми своего возраста, но все мы собираемся вместе для общественных мероприятий и обедов. Таким образом, разные члены семьи могут делать свой собственный индивидуальный выбор, но позже у них появляется возможность обменяться опытом.Наши центры предлагают первоклассное жилье, питание и удобства, а также первоклассный персонал. Квалифицированные учителя или профессионалы проходят у нас обучение, и многие из них работают с нами из года в год. У нас всегда есть квалифицированный персонал для занятий плаванием, прыжками на батуте и гимнастикой, но некоторые из помощников, организующих детские игры, — студенты, многие из которых сами приехали в лагерь, когда были моложе.

Вопросы и ответы на собеседовании с учителем начальной школы

Вот общие вопросы собеседования для учителей при подаче заявления на должность учителя начальной школы.Примеры ответов — это всего лишь предложения того, что может искать комитет по найму, но, очевидно, вам нужно будет подумать о своем опыте, прошлом и знаниях.

Комитет будет изучать вашу личность, и если вы подходите для команды учителей на уровне своего класса, будьте собой! 🙂 Комитет, скорее всего, высоко оценит тех, кто имеет несколько сертификатов в дополнение к Elementary Ed. (ESL, спец. Ред., Чтение..), а также двуязычных кандидатов, так что поделитесь всем, что вы можете предложить!

Изображение использовано с разрешения www.best-job-interview.com.

Подготовьте и принесите портфель

Интервью очень нервничают. Я настоятельно рекомендую взять с собой портфолио, которое покажет, кто ВЫ как учитель. Создайте один онлайн (создайте веб-сайт, PowerPoint или СОЗДАЙТЕ LIVEBINDER), принесите свой ноутбук на собеседование или сходите в ближайший офисный магазин и купите папку с тремя кольцами, защитные пленки и разделители.Портфолио должно включать ответы на следующие возможные вопросы собеседования, в частности:

  • Содержание
  • Резюме
  • Стандартное сопроводительное письмо
  • Все сертификаты ~ лицензия на преподавание, SEI, CPR, CPI, Orton-Gillingham. . .
  • Список всех клубов, комитетов и т. Д., В которых вы участвовали в качестве учителя и / или ученика-учителя ~ были ли вы представителем PTO, членом совета школьного сайта, членом климатической группы, членом группы данных, учителем-наставником новый учитель.. .?
  • Выписки
  • Полученные вами награды или награды
  • Свидетельства о пройденных вами занятиях / семинарах
  • Рекомендации бывших работодателей, письма довольных родителей, письма студентов
  • Ваша философия преподавания ~ абзац. Примеры приведены ниже.
  • Примеры планов урока ~ постарайтесь включать и ELA, и математику.Если у вас нет хорошего урока, сделайте два для той оценки, на которую вы собираетесь пройти собеседование! Вот планы уроков!
  • Отчеты о наблюдении за преподаванием от вашего бывшего директора или советника колледжа
  • Информация об учебной программе и / или оценке, которую вы знаете ~ Fountas & Pinnell Guided Reading, Engage New York, Lucy Calkins, Fundations, Pearson DRA2, текущие рекорды, Open Circle. . . Узнайте, в какую школу вы поступаете, и напишите об этом небольшую аннотацию, чтобы включить ее в свое портфолио!
  • Фотографии проектов ваших учеников и фотографии постановок ~ ставил ли ваш класс спектакль, спектакль или другой спектакль?
  • Фото собранных вами досок объявлений
  • Фотографии вашего класса, которые показывают ваши центры, распорядок дня, организационную систему! Вы даже можете зайти в Google Images и найти фотографии ИДЕИ, которые вы хотели бы реализовать в своем собственном классе! Например, вот идеи: центры грамотности, разговор по математике, организованный начальный класс, гибкие рассадки.Также посетите мою страницу в инклюзивном классе!
  • Напишите о важности того, чтобы ваши ученики развили УСТАНОВКУ РОСТА!
  • Примеры сотрудничества с коллегами ~ даты, ваши достижения, цели. Вспомните, когда вы сотрудничали с учителем речи и языка, специалистами, эрготерапевтом ~ как вы работали вместе?
  • Последние написанные вами научные статьи
  • Полевые поездки, которые вы спланировали и организовали
  • Примеры участия родителей ~ как то, что вы делали в прошлом, так и идеи на будущее ~ веб-сайт, информационные бюллетени, программы школьных представлений, таблицы домашних заданий и т. Д.
  • Примеры использования технологий ~ Эффективное электронное обучение
  • Технологии, которыми ВЫ владеете ~ Microsoft Word, PowerPoint, Excel, Adobe Illustrator, In-Design. . .
  • Примеры стратегий, которые можно использовать для полного вовлечения ваших ELL и студентов в IEP ~ создайте список стратегий! RETELL и стратегии чтения могут вам помочь! В качестве примера стратегии чтения поместите основную диаграмму предложения:
  • W Расскажите о различных типах оценок.Включите свой опыт в подготовке студентов к экзаменам с высокими ставками, таким как экзамены PARCC или MCAS 2.0
  • Общие основные смены и информация ~ специально для класса, для которого вы собираетесь пройти собеседование. Общее ядро ​​
  • Членство в профессиональных организациях или информационных бюллетенях, таких как Международная ассоциация грамотности или Национальная ассоциация образования. Если вы недавно закончили колледж, для вас будет неплохо присоединиться к таким организациям.
  • Информация о том, кто ВЫ вне школы. Каковы ваши таланты и интересы? Вы артист, музыкант, спортсмен. . . ?

Еще идеи для портфолио

Другие идеи портфолио В этой статье предлагается сделать 6 дубликатов «мини» портфолио ~ по одному для каждого интервьюера, чтобы просматривать их во время интервью! Это было бы полезно!

Приятной вдохновляющей цитаты на обложке вашего портфолио!

Общие вопросы с ответами

Ваша философия.. .

1. Какова ваша философия образования? (Эту аннотацию можно написать прямо в вашем портфолио!) Напишите анкету! Вот несколько идей:

  • Философия 1
  • Философия 2
  • Философия 3
  • ОБРАЗЕЦ: Моя философия образования состоит из нескольких частей. Эффективный учитель должен быть фасилитатором , чтобы руководить практическим обучением , а не просто лектором.Эффективный учитель должен стремиться развивать учеников на протяжении всей жизни и решать проблемы и продвигать самодисциплину и ответственность . Образование не только академическое — оно эмоциональное, социальное и физическое.

Что вам больше всего нравится. . .

2. Что вам больше всего нравится в преподавании? (Ваше портфолио будет говорить о вашей организации и творческих способностях, но также будет включать некоторые из следующих моментов.. .)

  • Работа с детьми
  • Удовлетворенность ростом учащихся во всех аспектах ~ в учебе, обществе, поведении.
  • Положительный вклад в развитие ребенка
  • Интеграция технологий в учебную программу
  • Творчество, организованность, эффективность
  • Сотрудничаем с коллегами
  • Помощь студентам в раскрытии их потенциала
  • Бросить вызов самому себе, постоянно изучая ~ всегда есть новый учебный план или новый метод, который можно попробовать.

Лучший урок. . .

3. Опишите лучший урок или раздел, который вы разработали и преподали. (Это будет у вас в портфолио!) Попробуйте провести урок ELA и математики. Укажите конкретно, чем на самом деле занимались студенты и как вы их оценивали. Подумайте о целях и стратегиях интерактивного обучения, которые были включены. RETELL содержит список интерактивных стратегий.

Общее ядро.. .

4. Поговорите об общем ядре и о том, как оно влияет на ваше обучение. (Это будет у вас в портфолио!) Common Core. . .

  • Ориентирован на готовность к колледжу и карьере
  • Студенты приводят доказательства
  • Студенты читают больше документальных текстов
  • Строгая учебная программа
  • Высокие стандарты и ожидания
  • Допрос на высшем уровне
  • Студенты копают глубже
  • Реализовать стратегии внимательного чтения

Ваши блоки ELA и Math.. .

5. Что мы увидим во время вашего блока математики на уровне класса, на который вы претендуете? Ваш блок ELA? (Это будет у вас в портфолио!)

  • Цели и стандарты Common Core четко размещены и обсуждаются со студентами.
  • Мини-урок с визуальными эффектами, использование всех модальностей ~ зрение, слух, движение, осязание (визуальное, слуховое, кинестетическое, тактильное)
  • Постепенное освобождение от ответственности модель
  • Весь класс, малая группа, партнер, индивидуальная работа
  • Использование интерактивных стратегий обучения, таких как Turn and Talk, Think / Pair / Share, Jigsaw, Numbered Heads Together ~ см. RETELL
  • Учащиеся, работающие на своем уровне, «с досягаемостью» для перехода на следующий уровень
  • Много похвалы и положительного подкрепления
  • Учитель перемещается по комнате, когда не работает напрямую с учениками
  • Учитель ведет учет, конференц-связь с группами и отдельными лицами
  • Учащиеся выполняют задание
  • Учащиеся могут выбирать места, задания и проекты
  • Каждый ребенок получает то, что ему нужно ~ справедливое не всегда равно
  • Студенты ELL и студенты по IEP полностью включены и участвуют
  • Задаваемые вопросы более высокого уровня
  • Использование методов, основанных на исследованиях (см. Меню ИССЛЕДОВАНИЕ на моей домашней странице)

Положительный климат в классе.. .

6. Как создать позитивный климат в классе? (Этот план будет у вас в портфолио!)

  • Быть позитивным человеком
  • Не повышаю голоса
  • Использование невербальных сигналов
  • Достоинство и уважение к студентам
  • Попросите учащихся создать правила в первый день, чтобы они владели правилами, ожиданиями и последствиями ~ публикуют правила, и учащиеся подписывают «контракт» с правилами.”
  • Последовательность
  • Наличие гибких сидений ~ подвижных стульев, мячей для йоги, сидений на полу с низким столиком, стоя за постоянным столом ~, если это разрешено в районе / школе.
  • Предоставление учащимся возможности выбора места для рассадки, проектов, заданий
  • Подчеркивая позитив
  • Реализация учебной программы «Открытый круг» или программы поведения в округе
  • Знать своих учеников и настраивать каждого на успех
  • Предотвращение плохого поведения за счет упреждения и избегания потенциальных ловушек
  • Использование невербальных сигналов для перенаправления учащихся
  • Распределение работы в классе, чтобы у учащихся было чувство гордости, собственности и принадлежности.
  • Цели и награды для всего класса, такие как день пижамы, обед как пикник на свежем воздухе. . .
  • Посещение отдельных внеклассных мероприятий студентов и демонстрация моей самоотдачи и заботы о каждом студенте
  • Позвоните домой или по электронной почте, чтобы сообщить ХОРОШИЕ новости!
  • Class DoJo ~ Исследуй это!

Дисциплина. . .

7.Как бы вы поступили со студентом, который не следует правилам поведения в классе? (Опять же, это будет у вас в портфолио!)

  • В беседе 1: 1 со студентом поинтересуйтесь, что может быть причиной такого поведения.
  • Подчеркнуть позитив
  • Настройте учащегося на успех ~ предоставьте учащимся варианты размещения в классе ~ нужен ли учащийся стол большего размера, частые перерывы, измененное задание.. .?
  • Внедрите дневник между домом и школой, чтобы родители могли участвовать и помогать перенаправлять поведение
  • Сообщить родителю ХОРОШИЕ НОВОСТИ
  • Имейте таблицу поощрений, чтобы ученик зарабатывал чтением вслух в детском саду, обедом с учителем или другой целью, к которой нужно стремиться. Держите карту в тайне между вами и учеником.
  • Обратитесь к консультанту по адаптации школы, чтобы узнать, могут ли потребоваться услуги SAC.
  • Соберите данные о поведении ~ что его вызывает? Что ты пробовал?
  • Если все остальное не помогает, обратитесь за советом в группу поддержки учителей, если в школе есть такой способ ~ таким образом, поведение официально задокументировано. Вы можете совместно с коллегами разработать план исправления.

Связь с родителями. . .

8. Как вы общаетесь с родителями? Как вы вовлекаете родителей в обучение? (Опять же, это будет у вас в портфолио!)

  • В первый день в школе отправьте домой опрос и соберите данные о том, как лучше всего связаться с каждым родителем — по телефону (и когда лучше всего звонить) или по электронной почте.Приложение «Напоминание» отличное!
  • Твиттер
  • Веб-сайт класса ~ размещение планов уроков, домашних заданий, способов помощи ребенку, ссылки на онлайн-ресурсы
  • Оценок можно разместить на PlusPortals. Использует ли округ онлайн-систему для родителей?
  • Школа или PTO Facebook могут предупреждать родителей о событиях
  • Информационные бюллетени для учебных заведений
  • Google Класс ~ учителя публикуют задания, чтобы родители могли их просматривать
  • Папка с домашними заданиями на день с указанием домашних заданий, которые нужно подписать
  • Домашний / школьный дневник поведения при необходимости
  • Родители будут приглашены на выступления в классе, в качестве приглашенных читателей или для 30-минутного обучения в той области, в которой они специализируются.
  • Родители могут оставить голосовую почту, и учитель перезвонит родителям
  • Снова в школу и другие школьные мероприятия
  • Утренние и дневные обязанности, которые учитель может столкнуть с родителями и быстро проверить их на работе
  • Переводчиков можно вызвать заранее для двуязычных конференций
  • Родители-волонтеры для выездов на места
  • Сообщать ХОРОШИЕ НОВОСТИ домой по мере необходимости

Как бы вас описали.. .

9. Как бы вас описали коллеги? (Подумайте о положительных прилагательных.)

  • Выделенный
  • Трудолюбивые
  • Доступен
  • Командный игрок
  • Вспомогательный

ELLS и IEP. . .

10. Что важно при работе со студентами ELL и со студентами по IEP? (Ваш список в вашем портфолио!)

  • Сопровождение размещения.
  • Необходимо знать уровни WIDA, а также цели и условия IEP
  • При необходимости изменить работу
  • Преимущественная посадка
  • Сделайте так, чтобы все дети почувствовали себя вовлеченными, желанными и успешными
  • Реализация интерактивных стратегий обучения RETELL
  • Обучайте, используя все 4 метода, чтобы охватить всех учащихся
  • Используйте совместную групповую работу, чтобы учащиеся могли учиться друг у друга и мотивировать друг друга
  • Храните данные и отслеживайте рост
  • Сотрудничать с коллегами

Оценка и размещение.. .

12. Как вы оцениваете успеваемость студентов? (Проверьте свое портфолио! 🙂)

Формирующее оценивание: Целью формирующего оценивания является мониторинг обучения учащихся для обеспечения постоянной обратной связи, которая может быть использована для улучшения преподавания и улучшения обучения студентов. Формирующие оценки помогают учащимся определить свои сильные и слабые стороны и целевые области, в которых необходимо поработать. Кроме того, они помогают учителям определить проблемные области, которые необходимо немедленно исправить.

Суммативное оценивание: Цель итогового оценивания — оценить работу учащегося в конце раздела, сравнив ее с каким-либо стандартом или эталоном. Это высокие ставки или высокая ценность.

Оценка успеваемости: Оценка успеваемости — это оценка, при которой учащиеся демонстрируют, что они овладели определенными навыками, выполняя или производя что-то.

Аутентичная оценка: Аутентичная оценка имеет 2 характеристики.Во-первых, они могут быть встроены в рутинные занятия в классе и обучаться и проводиться во время обычных занятий. Они явно связаны с обучением и достижениями, которые являются целями, и выводы, которые мы делаем из этой информации, могут быть связаны с обучением учащихся. Примером может служить школьный магазин: расставляет товары, считает деньги, раздает сдачу. Во-вторых, они могут быть сосредоточены на чтении из реального мира. Эти задачи сосредоточены на мире за пределами школы.

«Другой»: Оценка важных факторов, таких как мотивация, вовлеченность, самооценка, свобода действий, интерес и отношение.

Всегда анализируйте данные и используйте данные для руководства обучением. Часто перегруппируйте студентов на основе данных. Поговорите с прошлогодним учителем и со специалистами этого года, чтобы таким образом отслеживать успехи учащихся.

  • Текущие записи
  • DRA2 ~ это неформальная оценка чтения
  • Предварительные и последующие тесты
  • Модульные тесты
  • Тесты
  • Ежедневные наблюдения
  • Выходные билеты
  • Прогресс ANET, MCAS 2.0 баллов, баллы PARCC. . .
  • Оценки ~ почерпните некоторые идеи на этой странице.
  • Студенты могут самостоятельно оценивать. Вот несколько примеров самооценки и оценки коллег
  • Соберите студенческие работы для создания портфолио роста
  • Некоторые студенты могут быть официально аттестованы на предмет потенциальных IEP. При необходимости разместим:

Интервью с учителем начальной школы

Общие помещения

  • Четкие и краткие указания, правила и ожидания
  • Разбивка задач на более мелкие части
  • Установите зрительный контакт с учеником, прежде чем давать указания
  • Повторение и / или повторение направлений
  • Преимущественная посадка
  • Ограничение отвлекающих факторов
  • Использование наглядных пособий (расписаний, правил, задач)
  • Частые заезды
  • Моделирование языковых и учебных ожиданий
  • Просмотреть, уточнить и повторить новую информацию
  • Дайте студентам дополнительное время для обработки информации и дополнительное время для формулирования ответа.
  • Попросите учащихся повторить указания и инструкции, чтобы убедиться, что они понимают, о чем от них просят.
  • Свяжите новый материал с предшествующими знаниями
  • Использование графических органайзеров
  • Мультимодальный подход к обучению
  • Положительное армирование
  • Возможности перерывов в движении

Сотрудничество.. .

13. Как вы сотрудничаете с коллегами? Если бы вы раньше были учителем или помощником, встречались ли вы в еженедельных группах поддержки учителей, посещали ли строительные собрания, посещали собрания по учебной программе? Разберитесь с ними! Если вы недавно закончили колледж, расскажите о совместной учебной работе, которую вы проделали в колледже!

Мое личное сотрудничество включает в себя STAT (еженедельные собрания группы помощи ученикам и учителям), школьный совет ~ собирается ежемесячно для обсуждения общешкольных проектов и бюджета, LAT (группа по изучению языка) ~ встречается еженедельно для обсуждения роста изучающих английский язык, ежемесячные встречи группы по учебной программе и ежемесячные строительные собрания, я координирую работу досках объявлений в фойе на основе отзывов и идей сотрудников, мы используем формы Google в качестве сотрудников для совместной работы над целями и проектами студентов с помощью кнопки «Приглашение к редактированию», у нас есть встречи группы по работе с данными, на которых мы обсуждаем студенты, цели, уроки и перегруппировка студентов.. . .

Технологии. . .

14. Как вы используете технологии в классе? (Проверьте свое портфолио!)

  • Получите идеи для своего портфолио с моей страницы! Эффективное электронное обучение
  • СПРОСИТЕ КОМИТЕТУ ДЛЯ ИНТЕРВЬЮ: Какие технологии доступны для использования учащимися / учителями?

Сильные и слабые стороны. . .

15. Каковы ваши сильные / слабые стороны? Ваши сильные стороны:

  • Организация
  • Внимание к деталям
  • Опыт работы на занятиях ~ примеры успехов!
  • Стремление и мотивация продолжать посещать семинары и занятия
  • Общение с коллегами и родителями
  • Позитивный настрой
  • Личный опыт студента (поделитесь) и что побудило вас стать учителем

Ваши слабые стороны должны быть ПОЛОЖИТЕЛЬНЫМИ и действительно СИЛЬНЫМИ !!!

  • Напористость, решительность и настойчивость, которые не дадут вам уснуть по ночам, придумывая способы охватить всех учащихся и способы сделать обучение увлекательным, интерактивным и увлекательным.
  • Использование всех возможных методов и тактик для охвата всех учащихся.
  • Принимайте вызовы и никогда не отступайте от них. Вы действительно преуспеваете, когда у вас много дел, когда это приносит пользу учащимся.

Вопросы. . .

16. У вас есть к нам вопросы? (Проявите интерес к школе и должности ~ хотя и соблазнительно, не задавайте конкретных вопросов о дате приема на работу.) Вместо этого задайте пару вопросов, которые они задали ВАМ! Например:

  • Как классные учителя внедряют технологии на ____ уровне класса?
  • Какие программы по английскому языку и математике используются в этой школе?
  • Как эта школа привлекает родителей к обучению?
  • Есть ли в этой школе группа поддержки учителей? (Встреча команды для обсуждения учащихся, у которых могут быть проблемы в поведении или учебе ~ для консультирования учителей по устранению проблем.)
  • Какой тип профессионального развития предлагает / будет предлагать этот округ?
  • Какие виды реагирования на вмешательство принимает округ?

ЗАКЛЮЧЕНИЕ НАЧАЛЬНОГО ИНТЕРВЬЮ С УЧИТЕЛЕМ

  • Поблагодарите вас за уделенное время и скажите, что вы были бы признательны за возможность провести демонстрационный урок!
  • Предложите оставить портфолио на ночь для их просмотра или оставьте информацию о том, как они могут повторно посетить ваше онлайн-портфолио.
  • Пожмите руку комитету, если вы не сделали этого вначале.

Если вы думаете о любых других возможных вопросах и / или ответах на собеседование с учителем начальной школы, дайте мне знать! Я их добавлю!

Документ в формате Word на этой странице доступен после пожертвования в размере 10 долларов на оплату обслуживания моего веб-сайта. Нет графики из-за законов об авторском праве. Спасибо!

Олимпиада по истории 6. Задания к школьному этапу олимпиады по истории (6 класс).Как провести олимпиаду по истории России

Всероссийская олимпиада школьников по истории

Школьный этап

6 класс

2 балла

Задание 1. С какого года принято считать историю средневековья? 1. Отметьте правильный ответ.

A) 476 BC

B) 1 BC

C) 1 год нашей эры

D) 476 год нашей эры

2.Какое событие, ознаменовавшее конец истории Древнего мира, произошло в этом году?

______________________________________________________________________________________________________________________________________________

9 баллов

Z

A) Организация библиотеки глиняных книг

B) Отмена рабства в Афинах

C) Создание новой религии — ислама

D) Военный поход в Индию

E) Первый правитель государство франков

F) Создание единого свода законов для всей Византии

G) Захват единоличной власти в Риме

H) Перенос польской столицы в Краков

I) Открытие «Дворца» Академия »в Аахене

адания 2.Распределите имена правителей и их деятельность столбцами в соответствии с эпохами, в которых они жили — Древний мир и Средние века.

1. Александр Македонский

2. Мухаммад

3. Юлий Цезарь

4. Хаммурапи

5. Солон

6. Болеслав Храбрый

7. Юстиниан

8. Карл Великий

9. Хлодвиг

История древнего мира

3 балла

Задача 3.В средние века считалось, что образованный человек должен изучать определенные науки. Они получили название «Свободное искусство». 1. Сколько их было? _________________________________________________

2. Выбрать ненужное из предложенного списка:

A) риторика

B) грамматика

C) диалектика

D) литература

E) астрономия

E) музыка

G) геометрия

H) арифметика

I) 6000 точек

Задача 4.Составьте грамотное определение из слов, предложенных в правом столбце, и найдите слово в левом столбце, к которому относится это определение.

Условия

Введите результаты в таблицу:

5 баллов

Задание 5. В газете опубликована заметка следующего содержания: «В Калининградской области ____________ памятник герою Отечественной войны 1812 года русскому полководцу Михаилу Богдановичу Барклаю де Толли был изуродован.Памятник решили сдать на металлолом, для чего забросали его кувалдами и другими тяжелыми предметами, в результате чего памятник был сильно поврежден. «

Вместо пропущенного слова журналисты использовали название одного из германских народов.

1. Какое слово отсутствует?

2. Почему он используется в таких ситуациях?

________________________________________________________________________________________________________________________________________

4 балла

Задача 6.Какие «ступеньки» отсутствуют на феодальной лестнице?

1. Восстановить недостающее: 1________________________; 2____________________________.

2. Может ли феодал 2, давший присягу на верность феодалу 1, считаться вассалом короля? Почему? Какое средневековое правило действует здесь?

_____________________________________________________________________________________________________________________________________

1 2

2 балла

Задача 7.Предлагаемая мозаика изображает одного из величайших и самых могущественных правителей Византии и его жену. Соедините квадраты с буквами, чтобы получить имена этих исторических личностей.

Запишите их: 1 .______________________; 2 .___________________________.

Факс

Максимальное количество баллов: 31

Время подготовки ответа: 45 минут

Задания школьного этапа Всероссийской олимпиады по истории

2015/2016 учебный год

6 класс

Уважаемые ребята!

Мы рады приветствовать всех участников, проявивших интерес к Исторической олимпиаде! Поздравляем с началом школьного этапа Всероссийской олимпиады школьников! Мы уверены, что ваши таланты, ваш труд не только помогут вам в вашем выступлении на олимпиаде, но и в ближайшем будущем станут залогом успешного развития нашей страны.

Рабочая инструкция

На выполнение работы по истории, 1 час (60 минут).

Работа состоит из 7 заданий.

К заданию 1 нужно выбрать фразу.

К задаче 2 — дается 1 вариант ответа, требующий краткого ответа ( да или не ).

В задаче 3 нужно вывести расчеты.

В задаче 4 вам нужно исправить исторические ошибки.

Задачи 5 и 7 требуют только одного правильного ответа на поставленный вопрос.

В задании 6 поместите правильные ответы в таблицы A и B.

Внимательно прочтите каждое задание и предлагаемые варианты ответов, если таковые имеются. Отвечайте только после того, как вы поняли вопрос и проанализировали все варианты ответов.

Выполняйте задачи в том порядке, в котором они даны. Если задача вас беспокоит, пропустите ее. Вы можете вернуться к пропущенным задачам, если у вас будет время.

Один или несколько баллов начисляются за выполнение заданий различной сложности. Очки, полученные вами за выполненные задания, суммируются.

Постарайтесь выполнить как можно больше задач и набрать как можно больше очков.

Максимум баллов за работу — 87 .

ВСЕРОССИЙСКАЯ ОЛИМПИАДА ШКОЛЬНИКОВ ПО ИСТОРИИ. ШКОЛЬНЫЙ ЭТАП. 6 КЛАСС.

_________________________________________________________________________

Задача 1. По какому принципу формируются ранги (за каждый правильный ответ 4 балла, максимальный балл — 12)?

а) собака, коза, корова -__________________________________________________

б) душа, духи, жертва, молитва — __________________________________________

в) век, год, тысячелетие -________________________________________________

Задание 2. Верны ли следующие утверждения? («Ну нет»). Введите ответы в таблицу (2 балла за каждый правильный ответ, максимальный балл — 16)

    Евфрат — главная река Египта.

    Финиковая пальма — плод хлебного дерева.

    Египетские боги олицетворяли природу.

    Современный человек появился около 20 тысяч лет назад.

    Первые Олимпийские игры состоялись в 771 году нашей эры.

    Отцом Александра Македонского был Дарий III.

    Самой зловещей фигурой среди императоровI века до нашей эры был Нерон.

Задание 3 … Решать задачи.

А. Цезарь был убит в 44 г. до н. Э. сколько лет прошло с тех пор? Принесите свои расчеты

Б. Захоронение совершено в год смерти Спартака. Сколько лет было этому захоронению, когда археологи обнаружили его в 2015 году? Принесите их расчет (7 баллов за правильный ответ).

Назначение 4 … Текст содержит 13 исторических ошибок, которые подчеркнуты.

Далее нужно написать правильный ответ (за каждый правильный ответ 2 балла, максимальный балл — 26).

В 753 г. до н. Э. на берегу Тигр …………… Был основан Рим. Согласно легенде, первые два века Римом правил год 12 …… королей. После отставок …………….…. последний из них Тарквиния Смелый ………… .. в 509 году нашей эры в Риме была установлена ​​ демократия ………….. .

За короткое время маленькая страна превратилась в огромную империю. В течение года четыре ……… .. Пунический воин римлян с Корфу ………………… Средиземное море превратилось во внутреннее озеро римлян. Рим и гражданские войны не обошли стороной. Фактически они начались с принятия земельного закона Гай ………… ..Гракх. Восстание Спартака г. 74 г. ………… .. потрясло государство. Результатом гражданских войн стало установление в Риме самодержавия.Первый император года …………… .. Гай Юлий Цезарь стал Римом.

Под ударами славянских племени …………… 576 г ……… пало и Римская империя ……… ..… .. ……….… . ……….…. … Этим событием закончилась история Древнего мира.

Задание 5 … Поставьте крестик или другой знак в нужной ячейке (каждый правильный ответ –1 балл).

1. Определите дату окончания истории Древнего мира и начала истории Средневековья

A) 410 g.Б) 395 г. В) 476 г. Г) 500 г.

2 … Когда произошло Великое переселение народов?

А) IV-VII вв. Б) III-IV вв. В) I-II вв. Г) VIII-X вв.

3 … Укажите, пожалуйста, имя вождя, которого прозвали «бичом божьим» за то, что он опустошил Европу.

А) Хлодвиг; Б) Аттила; В) Аларик. Г) Карл Мартелл

4. Древние документы хранятся в

A) архивах B) музеях C) офисах D) сейфах

5. Письменные источники по истории средневековья в Европе (каждый правильный ответ –1 балл).

А) Календари Б) Летописи В) Евангелия Г) Летописи

№вопрос

Задание 6. Сгруппируйте отрасли экономики по их принадлежности. Запишите ответ в виде последовательности цифр в соответствующей строке. (Каждый правильный ответ — 1 балл, всего 12).

A) Передающая ферма

B) Экономика производства

1.сбор ягод

2. охота

3. разведение кошек

4. сбор грибов

5. рыбалка

6. земледелие

7. пчеловодство

8. огородничество

9. ткачество

10. сбор яиц

11. коневодство

12. кузнечное дело

A (Передающая ферма)

B (Ферма-производитель)

Задача 7.

Византийский историк В.И. В. писал: «Их никоим образом нельзя склонить к рабству или покорности в их собственной стране.Они многочисленны, выносливы, легко переносят жару, холод, дождь, недостаток пищи. Они ласково относятся к прибывающим к ним иностранцам и при необходимости защищают их … Нужные вещи хоронят в тайниках, ничего лишнего открыто не владеют … »

О каких людях пишет византийский историк?

Запишите ответ словами ____________________________________

За правильный ответ — 2 балла

Вот тестовые задания Всероссийской олимпиады школьного этапа по истории России для 6 класса с ответами на них.

Представленные задания можно использовать в 6 классе, предоставляя учащимся возможность лучше понять и обновить ранее пройденный материал, а также определить, насколько хорошо учащиеся готовы к олимпиаде.

1. Чему поспособствовал переезд главы Русской Православной Церкви из Владимира в Москву?

  • А) падение власти Московского княжества
  • Б) Возвышение Москвы
  • В) обострение религиозных противоречий
  • Г) объединение земель Русских

О походе киевского князя на Константинополь говорится в этом отрывке Повести временных лет?

В год 6415 (907).Я пошел ____ к грекам, оставив Игоря в Киеве; Он взял с собой множество варягов, и славян, и чудов, и кривичей, и Меру, и древлян, и радимичей, и полян, и северян, и вятичей, и хорватов, и дулебов, и тиверцев, известных как толмачи: все из них греки называли «Великой Скифией». И со всем этим он ехал _____ на лошадях и на кораблях; и было 2000 кораблей. И он пришел в Константинополь: греки закрыли суд, и город был закрыт. И _____ сошел на берег и начал сражаться, и он совершил много убийств в окрестностях города с греками, и они разрушили много комнат и сожгли церкви.И те, кто попали в плен, одни были вырезаны, другие истязали, некоторых расстреляли, а некоторых бросили в море, а русские причинили грекам много другого вреда, как это обычно делают враги … И ______ приказал ему солдат делать колеса и ставить корабли на колеса. И когда подул попутный ветер, они подняли паруса в поле и пошли в город. Греки, увидев это, испугались и сказали, послав _______: «Не разрушайте город, дань вам дадим, какую хотите» … И приказали _______ дать дань за 2000 кораблей: 12 гривен на человека, и на каждом корабле было по 40 человек… И он повесил свой щит на воротах в знак победы, и вышел из Константинополя …

  • А) Рюриковичей
  • Б) Олег
  • В) Аскольд
  • Г) Святослав

3. Какое произведение является памятником литературы XIV века?

  • А) «Повесть временных лет»
  • Б) «Жизнь Бориса и Глеба»
  • C) «Слово о законе и благодати»
  • Г) «Песня о Щелкане Дудентьевиче»

4. В ходе каких природных явлений произошло формирование государства у восточных славян?

  • А) В результате разложения феодального строя
  • Б) В результате разложения родоплеменного строя
  • C) В результате декомпозиции примитивной системы

5.Устраните лишнюю линейку.

  • А) Олег
  • B) Ягайло
  • В) Святослав
  • Г) Владимир

6. Деятельность князя Владимира была направлена ​​на:

  • А) усиление княжеской власти
  • Б) запрет контактов с Византией
  • C) уравнивание в правах язычников и христиан
  • D) ведение походов на Хазарский каганат

7. На съезде князей в Любечи в 1097 году было принято решение

  • А) Подать в ведение киевского князя
  • Б) Самостоятельное владение землями, переданными по наследству
  • В) Передача вечевой части власти в землях-волостях
  • D) Создать совет князей для решения спорных вопросов

8.Успенский и Архангельский соборы на территории Московского Кремля были построены в период правления:

г.
  • А) Александр Невский
  • Б) Долгорукий Юрий
  • C) Донской Дмитрий
  • Г) Калита Иван

9. Какое из следующих событий произошло в 1433 году?

  • А) Битва на р. Клязьма
  • Б) Присоединение Пскова к Московскому княжеству
  • В) Битва на Куликовом поле
  • Г) Созыв Земского Собора

10.Введение Дня Святого Георгия означало …

  • А) Ограничение личной свободы крестьян
  • Б) Установление годовой платы крестьян за проживание на земле феодала
  • В) Подтверждение права крестьян в любое время переходить от одного собственника к другому
  • Г) Предоставление крестьянам права поступления на военную службу

11. Кто составлял основную часть населения Руси?

  • А) горожане
  • B) дружинники
  • В) Крестьяне-общинники
  • D) ремесленники

12.Люди вместе охотились, занимались собирательством, заботились о детях, защищались от посторонних. Всего было учтено:

  • А) друзья
  • Б) соседи
  • В) сограждане
  • D) родственники

13. Медный век — это …

  • А) Палеолит
  • Б) Мезолит
  • C) Неолит
  • D) Энеолит

14. В каком году князь Олег совершил поход на Византию?

  • А) 907
  • Б) 908
  • В) 910
  • D) 911

15.Какое значение имело правление Ивана Калиты?

  • А) разрушил зависимость русских земель от Золотой Орды
  • Б) завершил процесс объединения русских земель
  • C) обезопасил Московское княжество от набегов Орды
  • D) заключил унию с Великим княжеством Литовским

16. Какое из следующих событий произошло в 1240 году?

  • А) Взятие войсками Батыя Козельск
  • Б) Взятие Киева войсками Батыя
  • В) Взятие Рязани войсками Батыя

17.Когда славяне поселились в Восточной Европе?

  • A) 40 тыс. Лет назад
  • B) в VII — VI вв. ДО НАШЕЙ ЭРЫ.
  • в) в VI — VIII вв.
  • D) в X веке.

Ответы на Всероссийскую олимпиаду по истории России 6 класс

В данной таблице представлены ответы на тесты по истории России для школьного этапа Всероссийской олимпиады

.
Номер вакансии Правильный ответ
1 Б
2 Б
3 D
4 Б
5 Б
6 И
7 Б
8 D
9 И
10 И
11 IN
12 D
13 D
14 И
15 IN
16 Б
17 IN

Материал, представленный в этом разделе, представляет собой примеры олимпиадных заданий, которые готовятся для подготовки учащихся 6-х классов к олимпиаде по истории России.Предлагаем школьникам использовать эти задания в качестве тренажера, который поможет им углубить и систематизировать знания, полученные на уроках.

Учителя могут копировать и распечатывать задания к тестовой олимпиаде на уроке. Проверить и оценить результаты шестиклассников можно за считанные минуты, ведь на все задания олимпиады в истории России даны ответы и решения.

Олимпиада по истории России, 6 класс
Скачайте задания, заполнив форму!

После ввода данных кнопка загрузки станет активной

Контрольные задания

1.К предпосылкам образования Древнерусского государства относятся:
A) необходимость отражения внешних врагов
B) тесные экономические связи между славянскими племенами
C) принятие христианства
D) ​​Великое переселение народов

2. Раб в Древней Руси — это:
A) член крестьянской общины, зависимый от князя
B) обанкротившийся член общины, попавший в долговую кабалу за ссуду
C) наиболее бесправная категория населения, близкая к рабам
D) ​​член сообщества, заключивший договор, который согласился жить и работать с мастером на определенных условиях

3.Изложение Н.М. Карамзина: «Этот князь, названный Церковью
Равноапостольным, заслуживает имени Великого в истории» — посвящено
А) Святослав
Б) Владимир
В) Ярослав Мудрый
Владимир Мономах

4. Свод законов в Древнерусском государстве назывался:
А) Салическая правда
Б) Русская правда
В) Стоглав
Г) Свод законов

5. Литературный памятник XII века, в котором содержался призыв к прекращению княжеской розни:
А) «Слово о полку Игореве»
Б) «Домострой»
В) «Слово об уничтожении земли русской». «
Г)« Обучение детей »

6.Ряд дат, связанных с борьбой русского народа против немецких и шведских рыцарей:
A) 1237, 1238
B) 1240, 1242
B) 1243, 1252
D) ​​1262, 1263

7. Одним из результатов правления Ивана Калиты было:
A) превращение Московского княжества в одно из сильнейших на Руси
B) избавление от ордынского ига
C) конец княжеской междоусобицы на Руси. Земля русская
Г) образование единого государства Российского

8.Формирование единой территории Русского централизованного государства в основном завершилось:
A) Василием I
B) Ivane III
C) Василием II Темным
D) ​​Иваном IV Грозным

9. Территория, на которой в середине XVI в. сохранилось управление Земским собором и Боярской Думой, оно называлось:
А) судьба
Б) опричнина
В) Земщина
Г) посад

10. Самая известная работа Андрея Рублева:
А) икона Донской Богородицы
Б) иконопись «Воинствующая церковь»
В) «Троица»
Г) роспись стен Благовещенского собора в г. Москве. Кремль

Открытые вопросы

Вопрос 1
В описании владений одного монастыря в X веке.говорят, что крестьянин имеет полный надел земли. Он платит за одну свинью, фунт льна, трех цыплят, 18 яиц. По обычаю, неделю она пасет свиней в лесу. Три дня в неделю в течение года он работает на приусадебном участке хозяйского поля, собирает урожай, а во время сенокоса косит стог сена, работает в усадьбе. Его жене приходится ткать льняную одежду.
Укажите, какие из перечисленных пошлин являются барщинными, а какие — оброком?

Вопрос 2
Расположите в хронологическом порядке
изобретений.
1) Очки
2) Стеклянное зеркало
3) Шахматы
4) Зубная щетка
5) Папирус
6) Вилка
7) Пушка

Вопрос 3
Установите соответствие между старославянским названием месяца и
современным названием.

Старославянское имя.
1.Серпен 2.Вересен 3.Березозол 4. Желе 5.Сечен 6. Липец 7. Трава 8. Свирепый 9. Падающие листья 10. Червь 11. Пыльца 12. Грудь

Современные названия
A. Январь B. Февраль C. Марч D.Апрель Д. Май Э. Джун Дж. Июль В. Август I. Сентябрь К. Октябрь Л. Ноябрь М. Декабрь

Вопрос 4
Внимательно прочтите отрывок из произведения русской литературы. Определите, какой отрывок взят из
: «… Перед законом благодать, а прежде его — истина. Образ
закон и благодать Агара и Сарры, работающих на Агара и бесплатно для Сары, работающих на обоих
, тогда это бесплатно, но понять, кого можно почтить! .

Добавить комментарий

Ваш адрес email не будет опубликован. Обязательные поля помечены *